Sei sulla pagina 1di 135

CHAPTER ZERO

SOME BASIC MATHEMATICS


TRIGONOMETRY
The Right angle Triangle

H
O

A
From Pythagoras Theorem, H 2 O 2 A2
In addition,

However,

Therefore,

O
,
H
A
cos , and
H
O
tan .
A

sin

sin

cos

tan

O
H
A
H

O tan
A

sin
cos

0.1

O2
A2 O 2 A2

H2 H2
H2
Using Pythagoras Theorem, we conclude that

Also,

sin 2 cos 2

sin 2 cos2 1
0.2
Though we have proved equations 0.1 and 0.2 for a right-angle triangle, they are indeed
true for any angle .

Next, we draw the graphs of the trigonometric functions:

sin
1.5
1
0.5
0
-0.5 0

90

180

270

360

90

180

270

360

90

180

-1
-1.5

cos
1.5
1
0.5
0
-0.5 0

-1
-1.5

tan
30
20
10

0
-10 0

270

360

-20
-30

Fig. 1.1: The graphs of sine, cosine and tangent of , 0 3600


If the y-axis of the sine function is shifted forward by 900, we shall obtain the graph for
cosine function. We can therefore say that
sin 90 0 cos .
0.3
Similarly, we infer that
cos90 sin

0.4

From equation 0.3, we note that cos is ahead of sin , or in other words, Sin lags
Cos by 900.
We recall that
sin A B sin A cos B sin B cos A

sin A B sin A cos B sin B cos A


cos A B cos AsosB sin A sin B
cos A B cos A cos B sin A sin B

Thus,

sin 90 0 sin 90 0 cos cos 90 0 sin


= 1 cos + 0 sin
Again, we arrive at the result in equation 0.3. You can easily verify that equation 0.4 also
holds true.

Note that

tan180 0 tan

cos360 0 cos

sin 360 0 sin


In other words, the Sine and Cosine functions repeat after every 3600, while the Tangent
function repeats itself after every 1800. We therefore say that Cosine and Sine functions
have a period of 3600, while the Tangent function has a period of 1800.

sin A B sin A cos B cos A sin B

cos A B cos A cos B sin A sin B


From which it readily follows that
tan A tan B
.
tan A B
1 tan A tan B
As an example,
tan 90 0 tan
0
tan90
1 tan 90 0 tan
cot
You can easily show that
tan A tan B
,
tan A B
1 tan A tan B
from which it follows immediately that
tan90 0 cot
From equations 0.5 and 0.6, we conclude that
tan90 0 tan90 0
tan A B

0.5

0.6

EXPONENTIAL FUNCTIONS
The series expansion of the exponential function ex is
x2 x3
ex 1 x


2! 3!
To get the expansion for e-x, replace x by x in equation 0.7
x 2 x3
ex 1 x


2! 3!

0.7

0.8

Adding equations 0.7 and 0.8 and dividing the result by 2 gives
e x ex
x2 x4
1


2
2! 4!
But the right hand side of equation 0.9 is the series expansion of coshx .
Hence,
e x ex
coshx
2
Equation 0.8 equation 0.9 yields
e x ex
x3 x5
x


2
3! 5!
e x ex
sinh x
or
2

0.9

The square root of the number 4 is


4 4 1 2i , where 1 i . i is an example of an imaginary number.
2
i ( 1) 2 1 ; i 3 i 2 i 1 i i
i 4 i 2 i 2 1 1 1 .
Higher powers of i just repeat this set of four numbers. Hence, we can write a sequence

i, 1, i, 1, i, .
We then notice that this sequence looks like that of sin , or cos , taken in steps 900
from 00 to 3600. Respectively, these are:
0, 1, 0, -1, 0, , and
1, 0, -1, 0, 1, .
Thus, we expect that there might be a relationship between the number i , and sin and
cos . This is indeed so.
Replacing x by ix in the expansion for e x , we get
(ix) 2 (ix) 3
x 2 ix 3
e ix 1 ix

1 ix


2!
3!
2! 3!
Similarly,
(ix) 2 (ix) 3
(ix) 2 (ix) 3
e ix 1 (ix)

1 ix


2!
3!
2!
3!
From equations 0.10 and 0.11, we obtain

0.10

0.11

e ix e ix
x2 x4
1


2
2! 4!
and

0.12

e ix e ix
x3 x5
x


2i
3! 5!

0.13

The right hand sides of equations 0.12 and 0.13 are, respectively, the series expansion
for cos and sin .
Therefore,
e ix e ix
cos x
2
e ix e ix
sin x
2i

Similarly,
e iax e iax
cos ax
2
e iax e iax
sin ax
2i

DIFFERENTIATION
(i) Polynomials
Given the function f ( x) ax n , where a is a constant, the differential of f (x) with respect
to x is nax n1 . We write
d (ax n )
nax n 1 .
dx
Examples: 1. Find the differential of 3x 2 with respect to x .

d
(3x 2 ) 2 3 x 21 6 x
dx

2. Differentiate t 4 3t 1 with respect to t.


d 4
(t ) 4 1 t 41 (1) 3 t 11 4t 3 3t 2
dt

(2) Exponential and Trigonometric functions


We can differentiate the expansion for an exponential function to get the differential of the
function. For instance,
Let f (x) e x 1 x

x 2 x3


2! 3!

Then, differentiating term by term, we get


3x 2

3!

x2
1 x
e x
2!

Indeed, the Maclaurin series for f ( x) e g ( x ) is

f (x) 0 1 x

[ g ( x)] 2 [ g ( x)] 3


2!
3!
[ g ( x)] 2 g ( x)
dg ( x)
f ( x) g ( x) g ( x) g ( x)
(where g ( x)
)
2!
dx

[ g ( x)]2
g ( x)1 g ( x)

2!

e g ( x ) 1 g ( x)

g ( x)e g ( x )

It should now be easy to differentiate trigonometric functions.


d e iax e iax iaeiax (ia)e iax
d

cos ax
dx
dx
2
2

e iax e iax
e iax e iax
1
a
, since i
2
2i
i
It is left to the student to show that
d
sin t cos t
dt
ia

INTEGRATION
(1) Polynomials
We have seen that

d
ax n nax n 1 , where a and n are constants. Multiplying both sides by
dx

dx gives
dax n nax n 1dx
Integrating both sides,
n
n 1
dax nax dx c ,
where c is an arbitrary constant, called the constant of integration.
Thus,
ax n na x n1dx c
That is,

n 1

xn
dx
c , n 0
n

This can also be written as

n
x dx

x n 1
c , n 1. Notice that we still put c, and
n 1

not c. This is because c is arbitrary.


Example: Integrate 4x 5 with respect to x.
5
4 x dx

4 x 51
2
c x6 c .
5 1
3

(2) Exponential and Trigonometric functions


Recall that
d f ( x)
e
f ( x)e f ( x ) .
dx
Then,
f ( x)
f ( x)
de f ( x)e dx c

Therefore,
f ( x)
f ( x)
f ( x)e dx e c
Example: Integrate e 2 x with respect to x.
2x
2x
2e dx e c
which means that
1 2x
2x
e dx 2 e c .
Trigonometric functions, being sums of exponential functions are easy to integrate as all
we need to do is to integrate the terms in the sum one by one.
Example: Integrate Sin t with respect to t.
e it e it
dt
sin w t dt
2i

1 1
1
e it
e it + c
2i i
(i )

1 e i t e i t
1
+ c cos t + c

CHAPTER TWO: VECTORS


A vector is a quantity that possesses both magnitude and direction. Thus, a vector makes
sense only if we specify a magnitude and a direction for it. In other words, we can write a
vector A as A = (A, ), where A is the magnitude of A and the angle it makes with the
positive x-axis.
A

A
Fig. 2.1

Resolution of Vectors
Given the vector in Fig. 2.2, we can obtain the x- and y-components as indicated,
respectively.

Ay

Ay

Fig. 2.2

Ax A cos ; Ay A sin

Thus, it follows that we can write A iAx jAy , where i and j are, respectively, unit
vectors in the x- and y-directions. In another notation, we can write A ( Ax , Ay ) . For
example, the vector 5 units in the direction 53.13 can be written as (5, 53.130). On the
other hand, we can write A as Ax (5 cos 53 .13 0 , 5 sin 53 .13 0 ) (3, 4) .
On the other hand, suppose what we had was ( Ax , A y ) , we could get ( A, ) , in the
triangle above (Fig. 1):
A Ax2 Ay2

tan 1

(Pythagoras theorem)

Ay

Ax
As an example, let us consider the vector (3, 4) we got above, we could easily get
A = Ax2 Ay2 32 4 2 25 5
4
3

tan 1 = 53.130
Exercise: Fill in the missing spaces.

A
6

Ax

Ay

32
460

2.3

2
3.4 1.2

Addition of Vectors
The addition of vectors, we expect, would be different from the addition of scalars,
bearing in mind that vectors are directed quantities. We shall look at different methods of
adding vectors.
Graphical or Geometrical Method
Consider the vectors A, B, C and D as shown below.
A

B
C

Fig. 2.3

The vector A + B is got by first drawing A and then drawing B from the tip of the drawn
A. Notice that the resultant vector is drawn from the base of the first vector to the tip of
the second vector, because, in effect, you moved along vector A, and then vector B to
arrive at what would have been equivalent to moving from the same initial point (the base
of A) to the tip of A + B. The vector B + A is obtained by first drawing vector B and then
drawing vector A from the tip of the drawn B, as shown in Fig. 2.4b.
B
B+A
A
B
A+B
B
Fig. 2.4a
Fig. 2.4b
Notice that A B B A . Note, also, that the vectors are drawn to scale, meaning that
halving one vector means we would have to halve the others as well.
What about A B and B A ?
-B
A -B

B
B-A

Fig. 2.5a

A
Fig. 2.5b

Clearly, A B (B A) as should have been expected.


We could also see what A B C D is:
B
A

C
D

A+B+C+D
Fig. 2.6

The student can try A B C D on his or her own.


What happens when the vectors go completely round, that is, in Fig. 2.6, suppose the fifth
vector is indeed reversed. Then, the resultant vector is zero, 0. Note that the zero vector is
still written in boldface, as it is a vector of zero magnitude, and indeterminate direction. It
is not the same as 0, which is a scalar. Thus, the following figure shows that
a b c d e 0 . It means that if all these vectors are forces (the only forces) acting on
a body, the resultant force is zero, meaning that the body must be in equilibrium. This is a
generalisation of the law of triangle of vectors, which states that:
When a body is in equilibrium under the effect of three forces, we can
represent the forces by the three sides of the triangle, taken in order.
In this case, we can represent the forces by the sides of a polygon, taken in order.
c

b
e
a
Fig. 2.7

Parallelogram Law
The student is perhaps more familiar with this method than he or she is with any other,
being the mainframe of the O Level treatment of addition of vectors.
Let us once again draw vectors A and B as shown in Fig. 2.32. Then, we can write
B
A
P

R=A+B
R

Fig. 2.8
Note that R A B A B .
The fact that A B B A lends credence to the idea of representing the sum in a
parallelogram.
Let us consider the parallelogram. From cosine rule, we know that
R 2 A 2 B 2 2 AB cos(180 )
But we know that cos(A B) cos A cos B sin A sin B .
It follows that cos(180 ) cos180 cos sin 180 sin cos . Therefore,
R 2 A 2 B 2 2 AB cos
Considering the right angle triangle, then we can write,

10

A sin
B A cos
Where is the angle the resultant makes with the x -axis.
tan 1

Example
Find the resultant of the two vectors shown in Fig. 2.9.
5 units

600
4 units
Fig. 2.9

R 4 2 52 2 4 5 cos600 = 7.8102 units


5 sin 60 0
tan 1
= 33.670
4 5 cos 60 0
Addition via Resolution of Vectors
We could have solved the problem in Fig. 2.8 with the help of resolution of vectors. We
repeat the diagram here for effect.
A

Sum

y
0
A sin
A sin

x
B
A cos
B A cos

We could represent this with a right angle triangle

B A cos

A sin

R 2 ( B A cos ) 2 ( A sin ) 2 = B 2 A2 cos2 2 AB cos A2 sin 2

= B 2 A 2 (sin 2 cos 2 ) 2 AB cos A 2 B 2 2 AB cos


A sin
tan 1
B A cos
We solve the problem in Fig. 8 to compare with the method of parallelogram.

11

5 units

4 units

Sum

y
0
5 sin 600
5 sin 600

x
4
5 cos 60 0
4 5 cos600

R has the same formula, and so does


But which method is better? The second one of course, because you could add as many
vectors as possible, quite easily. On the other hand, to add 5 vectors using the
parallelogram law, you need to draw 4 parallelograms, and each time you would need to
calculate the angle the current resultant makes with the positive x-axis; quite cumbersome.
Example
Add the following vectors:
6.5 N

6N
1000

300

1500

5.5 N
x
5 cos30 0
6.5 cos1300
5.5 cos 2800

y
5 sin 300
6.5 sin 1300
5.5 sin 2800

4.3301
-4.1781
0.9551
1.1071

R (1.1071) 2 (2.0629) 2 2.3412 N


2.0629
tan 1
61.780
1.1071
Exercise
Add the following vectors

12

2.5
4.9793
-5.4164
2.0629

105

350

4m

7m
8m

13

CHAPTER THREE: KINEMATICS


Kinematics is the study of motion without making recourse to the forces causing it.
Central to kinematics are the concepts of displacement, velocity and acceleration.
The displacement of a body is the distance it moves in a specified direction.
Displacement is measured in m.
The velocity of a body is time-rate of change of its displacement.
Velocity is measured in m/s.
The acceleration of a body is the time-rate of change of its velocity.
Note that velocity and acceleration are time-rates. It is not sufficient to say that
dv
dv
acceleration is the rate of change of velocity. We must be specific, we mean
, not
,
dx
dt
which is also a rate of change, a spatial rate of change, if x is a measure of distance!
Rectilinear Motion
Rectilinear motion is motion in a straight line. In this case, the acceleration can only be
zero, positive or negative as there is only one direction. Consequently, velocity and
displacement can only be in the same or opposite direction.
Suppose the acceleration of a body is a . Then, we can write
dv
2.1
a
dt
Hence,
2.2
dv adt
Integrating both sides,

v0

dv adt

2.3

where the velocity at time t 0 is v 0 .


Therefore,
t

v v0 adt

2.4

or

v v0 adt

2.5

Example
Given that the acceleration of a body in m/s2 is a 3t 2 , find its velocity at time t 5 s,
if the initial velocity is 2 m/s.

14

Solution
t

3t 2

3
3
2t v0 t 2 2t 2 t 2 2t
v(t ) v0 adt = v0 0 (3t 2)dt v0
0
2
2
2
0
To get the velocity at time t = 5s, we substitute 5 for t.
3
v(5s) 2 (5) 2 2(5) 49.5 m
2
t

The expression for displacement can be got from equation 2.5.


t
dx
= v0 adt
0
dt
Hence,
t
dx v0 adt
0

Integrating both sides,


x
t
t
x0 dx 0 v0 0 adt dt
where we have assumed that the displacement at time t = 0 is x 0 .

2.6

2.7

2.8

Hence,
t
t
t
2.9
x x0 v0 dt adt dt
0
0 0

Therefore,
t
t
2.10
x x0 v0 t adt dt
0 0

We cannot proceed beyond this point, unless we know the dependence of a on t .

Examples
1.
Uniform motion
In uniform motion, the acceleration is zero, since the velocity is constant. Then, from
equation (2.10),
2.11
a0
v v0
2.12
x x0 v0 t
2.13
2.

Uniformly accelerated motion


a is a constant
t

2.14

v v0 adt v0 a dt v0 at

2.15

t
t
t
t
t
t
x x0 v0 t adt dt x0 v0 t a dt dt x0 v0 t at 0 dt
0 0
0
0
0

1
1
= x0 atdt x0 v0 t at 2 x0 v0 t at 2
0
2
2
0
Thus, the equations of uniformly accelerated motion are,
t

15

2.16

a0

v v0 at

1
x x0 v0 t at 2
2

Alternatively, from (2.15),


v v0 at
dx
v
dt
dx (v0 at )dt

dx (v

2.17

2.18
2.19
2.20

at )dt

2.21

1 2
2.22
at c
2
At t 0 , x x0 . Thus,
x0 c
Hence,
1
2.23
x v0 t at 2 x0
2
Example
The acceleration of a body is 1.4 m/s2. If its initial position is 3.2 m, and the initial
velocity is 2 m/s, what is its velocity and position at time t = 4 s?
x v0 t

Solution
v(t ) v0 at 2 (1.4)( 4) 7.6m / s
1
1
x(t ) x0 v0 t at 2 3.2 2(4) (1.4)(4) 2 22.4 m
2
2
Motion in More than One Dimension
For motion in two directions, say x - and y -directions, we write a ( a x , a y ) . Then, we
can treat each of these directions as if there is motion only in that direction.
Example
The acceleration of a body is given as (3, 4) m/s 2. If its initial velocity is (-1, 2) m/s, and
its initial position is (0, 0) m, find its velocity and position at time t = 3 s.
Solution
This is uniformly accelerated motion in two dimensions.
v v 0 at
Note that we have used the vector notation.
v (1, 2) (3, 4)t (1 3t ,2 4t ) (1 3[3], 2 4[3])
= (8, 14) m/s

16

2.24

1
1
x x 0 v 0 t at 2 (0, 0) (1, 2)(3) (3, 4)(3) 2
2
2
= (3 13.5, 6 18)
= (10.5, 24) m

Projectile Motion
Projectile motion is motion in a plane (an example of two dimensional motion). In this
case, the acceleration a (0, g ) m/s2, where g is the acceleration due to gravity, which
for the purposes of this class shall be taken to be numerically equal to 10 m/s2. The motion
is uniformly accelerated in the vertical direction ( y -direction) and uniform in the x direction.
y
v0 y

v0 x

R
Fig. 3.1
Then,

v v 0 at (v0 cos , v0 sin ) (0, g )t = (v0 cos , v0 sin gt )


1
1
x x 0 v 0 t at 2 (0, 0) (v0 cos , v0 sin )t (0, g )t 2
2
2
where we have taken x 0 (0, 0) .
Therefore,
1
x ( x, y) (v0 t cos , v0 t sin gt 2 )
2

Instantaneous velocity
v x v0 cos , v y v 0 sin gt
v v x2 v y2 v02 cos2 (v0 sin gt ) 2

v02 cos2 v02 sin 2 g 2 t 2 2v0 gt sin

v02 (cos2 sin 2 ) g 2 t 2 2v0 gt sin

v02 g 2 t 2 2v0 gt sin

The student should be able to obtain the angle the velocity vector makes with the positive
x-axis.

17

Instantaneous position
x v0 t cos , y v0 t sin

x2 y2 =
=

1 2
gt
2

v02 t 2 cos 2 (v0 t sin

1 2 2
gt )
2

v02 t 2 cos 2 v02 t 2 sin 2

1 2 4
g t v 0 gt 3 sin
4

1 2 4
g t v 0 gt 3 sin
4
1
1
v0 t sin gt 2 v0 sin gt
y
2
2
tan 1 tan 1

x
v0 t cos
v0 cos
This is the angle the position vector makes with the positive x-axis.

v 02 t 2

Maximum height attained


The maximum height is attained where v y 0 , or
v0 sin gt1 / 2 0

or
t1 / 2

v 0 sin
g

v0
, which is the familiar
g
time for maximum height for a projectile launched vertically upward.

If 900 , then the maximum height is attained in time t1 / 2

Thus, the maximum height attained,


H y (t1 / 2 ) v0 t1 / 2

v sin 1 v0 sin
1
g

gt12/ 2 v0 0
2
g 2 g

v 02 sin 1 v 02 sin

g
2
g

v 02 sin
2g

This is maximum, for a given v 0 and g , when sin 1 or 900 . Then, H

v 02
. Note
2g

that this then boils down to vertical motion of the body.


Total time of flight
The projectile moves in the x -direction as if the motion in the y -axis does not matter.
That is, until the motion in the y -direction brings it down. Thus, the time that y 0
marks the total time of flight.

18

y (t ) v0 t sin

1 2
gt 0
2

Factoring,
1

t v0 sin gt 0
2

meaning that
1
t 0 or v0 sin gt
2
2v sin
Hence, t 0 , or t 0
=T
g
t = 0 coincides with the time the projectile was launched, while T is the total time of
flight. Note that T 2t1 / 2 . This is because the motion of the projectile is a perfect
parabola, since we are neglecting air resistance; the motion is symmetrical about the point
of maximum height.
The range is then,
2v0 sin 2v02 sin cos
x(T ) v0T cos v0 cos

g
g
But sin 2 sin( ) sin cos sin cos 2 sin cos . Hence,
R x(T )

v02 sin 2
g

When 900 , R 0 , as is to be expected, since the motion will be vertical. The


maximum range is obtained when sin 2 1 or 450 .
Equation of the path
The x and y components of the displacement are given by
1
x v0 t cos , y v0 t sin gt 2
2
Clearly,
x
t
v 0 cos
Putting this in the expression for y , we obtain
x
1
x

y v0 sin

v0 cos 2 v0 cos

or

y = x tan

1
x2
2
2v cos
2
0

This is in the form y bx cx 2 , where b tan and c

1
are constant.
2v cos 2
2
0

y bx cx 2 is the equation of a parabola. Thus, the path of the projectile is indeed a


parabola.

19

Example
1.
A projectile is launched with an initial velocity of 35 m/s at an angle of 400 to the
horizontal. Find:
(a) the velocity at time t = 2 s,
(b) the position at time t 2 s,
(c) the maximum height attained,
(d) the total time of flight,
(e) the range.
Solution
v0 35 m / s , 400
(a)
Velocity at time t 2s
v x (t 2s ) v0 cos 35 cos 40 0 26 .8116 m / s

v y (t 2s) v0 sin gt 35sin 400 10(2) 2.4976 m / s


Hence, v(t 2s) (26.8116) 2 (2.4976) 2 26.9277 m / s
Alternatively,

v v02 g 2 t 2 2v0 gt sin =


= 26.9277 m/s
(b)

352 (10 2 )(2 2 ) 2(35)(10)(2) sin 400

Position at time t = 2s.


x(t 2s ) v0 t cos (35 )( 2) cos 40 0 53 .6231 m
1
1
y(t 2s) v0 t sin gt 2 (35)(2) sin 400 (10)(2 2 ) 24.9951 m
2
2

x 2 y 2 (53.6231) 2 (24.9951) 2 59.1624 m


Alternatively,
1
1
x v02 t 2 g 2 t 4 v 0 gt 3 sin
4
2
= (352 )(2 2 ) (0.25)(10 2 )(2 4 ) (35)(10)(23 ) sin 400
= 59.1624 m
(c) (e) is left as an exercise to the student.
2.

Find the velocity a projectile should be launched in such a way as to make it pass
through the window (5 m high) of a building 6 m away from the point where it was
launched. Assume that the angle of launch is 550.
Solution
550, x = 6 m, y 5 m.
1
y x tan 2
x2
2
2v 0 cos

20

x tan y
v02

x2
2v02 cos2

62
x2
=
= 15.3305
2 cos2 ( x tan y) 2(cos2 550 )(6 tan 55 5)

v0 15.3305 3.9154 m / s
x2
62
0

6
tan
55

8.5689 3.5689 = 5
2v02 cos2
2(3.9154) 2 cos2 (55)
Uniform Circular Motion
Curvilinear motion is motion on a curve. A body is said to be in circular motion if its path
is a circle or an arc of a circle. A body is in uniform circular motion if it is in circular
motion, and in addition, the magnitude of its velocity (that is, its speed) remains constant.
Such motion is not uniform motion, because in uniform motion, the velocity (both
magnitude and direction) are constant. A body in uniform motion cannot move on a curve.
So, here we shall be discussing uniform circular motion.
Check! y x tan

Radian measure
Before we study uniform circular motion in detail, it is imperative that we learn about the
radian measure. We are quite familiar with the idea of the circumference of a circle of
radius r being 2 r . Thus, 360 0 2 r
180 degrees is equal to radians, written as c . Thus, 1c
10

180 0

57 .30 . Conversely,

0.01745 c . Thus, we could measure angles in either degrees or radians. What


180
advantage does measuring in radians have over the measuring in degrees? Back to the
circumference of a circle again.
360 0 2 r

2 r
0
0


2 r = length of the arc subtended by angle 0
0
0
360
360
But suppose we measure angles in radians. Then,
2 r
c
c
c
2 r r c = length of the arc subtended by angle c
2
2
Thus, when we measure angles in radians, the length of the arc is just the radius multiplied
by the angle subtended, hence s r c in Fig. 3.3.
0

Consider a body moving on a curve as shown in Fig. 3.3.


r

Fig. 3.3

21

s r c

At time t 0 , r lies on the dotted line. At some other time t , it makes an angle of with
the reference line. t , where is the angular frequency of the rotation.
s r c

The linear velocity is given by

ds
. Therefore,
dt

ds d
(r c )
dt dt
But for circular motion, r is constant. Hence,
d c
vr
r
dt
d c
is called the angular frequency or angular velocity or angular speed,
where
dt
measured in radians per second (rad/s) or revolutions per minute, the former being the
standard unit. We can easily convert from one unit to the other, however, by bearing in
mind that 1 revolution is 2 c .
v

Example
Convert 5 rad/s to rev/s.
Solution
1 rev/s = 1 2 c / s 2 rad/s
1
1 rad/s =
rev/s
2
1
5 rad/s = 5
rev/s = 0.7958 rad/s
2
Example
Find the angular frequency of an ant moving along a circle of radius 30 cm if it covers 1 m
in 30 s.
Solution
r = 30 cm, s = 1 m, t = 30 s.
s 1m
v
0.0333 m / s
t 30s
v 0.0333 m / s
0.0111 rad / s
v r , so
r
0.3 m
Radial or centripetal acceleration
Even though the speed is not changing, the direction is. Thus, the velocity in uniform
circular motion is not constant. Therefore, there is a kind of acceleration associated with
uniform circular motion. This acceleration is called the radial or centripetal acceleration,
and it has to do purely with the fact that the body is moving on a circle. The centripetal
acceleration is always pointing to the centre of the circle.
Consider Fig. 3.4.

22

vB

vB

vA

vA

Fig. 3.4
v A and v B have the same magnitude v . Then, if the angle between them is , then, v
is the length of the arc indicated in Fig. 3.4. The centripetal acceleration is
d
d
a=
(since v is constant)
(v ) v
v
dt
dt
2

v
v
But v r . Therefore, a ( r ) 2 r = r
r
r
Example
Find the centripetal acceleration of the ant in the example above.
2

Solution
a = 2 r (0.0111 rad / s) 2 (1 m) 1.2321 10 4 m / s 2
Non-uniform Circular Motion
In this case, the radius of motion is constant, but the speed is no longer fixed. Thus, in
addition to s r c and v r and a 2 r , we need to add the angular acceleration, .
Here, the angular frequency is no longer constant.
d

dt
Measured in rad / s 2 , this acceleration produces a change in speed along the tangent to the
circle, enabling the body to slow down or speed up. If is constant, we get a set of
equations similar to the ones for linear motion, for angular motion.
Linear Motion
a
v0 , v
x
dv
a
dt
dx
v
dt
v v0 at

Angular Motion

0 ,

v 2 u 2 2as

2 02 2
1
0 t t 2
2

d
dt
d

dt
0 t

1
x v0 t at 2
2

23

Example
A flywheel rolls from rest with a constant angular acceleration of 2 rad / s 2 . Find its
angular speed and angular displacement at t = 1.5 s.
Solution
= 2 rad / s 2 , 0 0 , 0 0 , t 1.5 s .
0 t 0 2(1.5) 3 rad / s
1
1
The angular displacement 0 0 t t 2 0 (0)(1.5) (2)(1.5) 2 = 2.25 radians.
2
2

24

CHAPTER FOUR
DYNAMICS
Dynamics is the study of motion with reference to the force causing it. Central to the
concept of dynamics are linear momentum and the laws of motion due to Newton.
Linear Momentum, Impulse
The linear momentum of a body is the product of its mass and its velocity.
Thus, if the mass of a body is m and its velocity is v, then its linear momentum is
p mv . Note that linear momentum is a vector, since the product of a scalar and a vector
is a vector.
The change in the linear momentum of a body is the impulse it develops. Thus, if the
velocity of a body changes from v 0 to v , then the impulse is m( v v 0 ) , where m is the
mass of the body. Yet again, the impulse developed by a body is a vector quantity.
Newtons Laws of Motion
1st Law
Newtons first law of motion states that a body at rest or in a state of uniform motion
continues in that state until (unless) an external force acts on it.
Explanation
This law states that a body not acted upon by an external force would remain at rest or if it
was already in motion, would maintain the velocity. This law indeed recommends some
special frames of reference (coordinate systems in simple terms) in which a body at rest or
in a state of uniform motion will have no acceleration. Not all frames of reference fit into
this. The ones that fit into it are called inertial frames.
Some students are fond of stating this law as uniform motion in a straight line
However, uniform motion itself implies motion in a straight line, because any motion on a
curve is already suffering acceleration, even if the magnitude of the velocity is not
changing.
2nd Law
The time-rate of change of the linear momentum is proportional to the applied force.
Explanation
Note that it is a time rate, and change of linear momentum, and not just the rate of change
of momentum. Later, we shall come across angular momentum. The student, therefore, has
to distinguish between the two.
d
F p
4.1
dt
However, the constant of proportionality is taken as unity. Therefore,
d
F p
4.2
dt
Written as

25

d
d
dv
dm
dm
= ma v
p (mv) m
v
dt
dt
dt
dt
dt

4.3

we see that F ma , unless the mass is constant. Examples abound, in which the mass of a
body is not constant. We readily remember a conveyor belt and a rocket in flight.
However, unless otherwise stated, we shall assume that the mass of a body is constant.
Newtons 2nd law and the conservation of linear momentum
dp
From F
, we notice that if this equation refers to a body or a system of bodies, then if
dt
the external force F is zero, then p = constant. This is the law of conservation of linear
momentum. For a single body, it means that it stays at rest or continues to move in a state
of uniform motion, if it is already doing so.
For a system that is constituted by two or more bodies, it means that the total linear
momentum of the system remains constant. Let us consider the case where there are only
two bodies m1 and m 2 , with initial velocities v 01 and v 02 and final velocities v 1 and v 2
respectively. Then, the initial momentum of the system is
p 0 m1 v10 m2 v 20
4.4
The final momentum of the system is
p m1 v1 m2 v 2
4.5
If there is no external force, then,
p p0
4.6
or
m1 v1 m2 v 2 m1 v10 m2 v 20
4.7
It follows therefore, that
m1 ( v 1 v 10 ) m2 ( v 2 v 20 )
4.8
This means that the change in the linear momentum of one body is equal and opposite to
the change in the linear momentum of the other body.
Example
A gun-bullet system has zero total momentum before the gun is fired, but as soon as it is
fired, the momentum gained by the bullet in moving forward is lost by the gun which
moves backwards, as the total momentum must again be zero.
3rd Law
Action and reaction are equal and opposite.
Explanation
Action and reaction forces act on different bodies. Suppose you are trying to pull a cart
with a rope. The cart pulls you back with the same force. You exert a force on the rope,
which in turn applies the same force to the block. The block pulls back on the rope with
the same force. The force on rope by block and the force on rope by you are not an actionreaction pair because they act on the same body, the rope. However, the force exerted by
you on the rope and the one exerted by the rope on you is an action-reaction pair. Likewise
the force exerted by rope on block and that of block on rope. Note that all the forces in this

26

case are equal, until the block begins to move. In that case, the force the man exerts on the
rope is more than the force the block exerts on the rope. There is a net force in the
direction of the rope away from the block, towards you.
FRM
FRC
FMR

FCR

Fig. 4.1
If the block is not moving:
The action-reaction pairs are: FCR and FRC ; FRM and FMR .
FMR and FCR are not an actionreaction pair even though they are equal, as the block is
not moving!
Application of Newtons Laws
Example 1
A uniform rectangular picture of mass m kg hangs from a nail on a wall as shown in Fig.
4.2a. Find the tension in the string.
T

W = mg
W mg
Fig. 4.2a

Fig. 4.2b

If the nail is not moving, then from Newtons first law, the net external force on it zero.
Then we can draw a free-body diagram of the forces acting on it.
Clearly,
2T sin mg
4.9
Therefore,
mg
T
4.10
2 sin

27

Example 2
Consider a block of mass m resting on a smooth surface (no friction) as shown in Fig.
4.3a.

m1

T
T

m2

Fig. 4.3a

m1g

Fig. 4.3b

m2 g
Fig. 4.3c

Fig. 4.3b shows the forces acting on the mass m1 . Since the block is not breaking the
surface on which it rests, that surface must be exerting an equal and opposite force on it
(Newtons third law). Hence,
m1g W
4.11
The tension in the string, T, tends to move the body in the positive x -direction. Hence, in
the x-direction, we can write (Newtons second law),
T m1 a1x
4.12
Fig. 4.3c depict the forces on the mass m 2 . The forces acting on it are m 2 g , its weight,
and the tension in the string, which is again T , since the same string links the two blocks.
But the forces acting on m 2 are in the y-direction. Hence (again, courtesy of Newtons
second law),
m2 g T m2 a 2 y
4.13
Putting equation 4.12 in equation 4.13,
m2 g m1 a1x m2 a 2 y

4.14

However, a1x a 2 y a .
Therefore,
m1a m2 g m2 a
Or,
m2
g
a
m1 m2

4.15

4.16

From 4.13,

mm
T m1 a 1 2 g
m1 m2
Thus, if m1 m2 m , a g / 2 , and T mg / 2 .

28

4.17

Example 3 (Atwood Machine)


Atwood machine consists of a fixed pulley with two masses hanging on either side, from a
single string (Fig. 4.4). We assume the masses have a motion down the right side.
T

m1

m1g

m2 g

m2

Fig. 4.4a

Fig. 4.4b

Fig. 4.4c

Considering mass m1 ,
T m1 g m1a
Considering mass m 2 ,
m2 g T m2 a

4.18
4.19

Adding equations 4.18 and 4.19,


(m2 m1 ) g (m1 m2 )a
Or
m m1
g
a 2
m1 m2

4.20

4.21

From 4.18,
m m1

m m1

g g m1 2
T m1 (a g ) m1 2
1 g

m1 m2

m1 m2
m m1 m1 m2
2m 2
g m1
g

= m1 2
m

m
m

m
m

m
2
1
2
2
1
1

2m1 m2
g
T =
m1 m2

4.22

If m1 m2 m , then a 0 (as is to be expected), and mg .


Example 4 (The Elevator)
In Fig. 4.5, a man of mass m is in an elevator. Suppose
Fe

Fig. 4.5

29

mg

Assume first that the elevator is accelerating upwards, that is, its velocity is increasing
upwards. We shall take this direction of acceleration (upwards) as positive. The mans
weight (true weight) is mg , acting downward. The force the floor exerts on him upwards
is Fe . This is his apparent weight in the elevator. Thus, we can write,
Fe mg ma
4.23
Thus, the apparent weight,
Fe m( g a)
4.24
Thus, the man feels heavier when the elevator is accelerating upwards.
What happens when the elevator slows down on its journey upwards? The acceleration
becomes negative upwards, since the velocity is reducing. Then, from 4.24,
Fe m( g a)
4.25
The man feels lighter as the elevator slows down.
For the lift accelerating on its trip down, we can still use equation 4.24, but with a now
negative, meaning that equation 4.25 applies. When the lift is slowing down to a halt on its
way down, then the acceleration is again upwards (positive), opposing the velocity which
in this case is downward. Equation 4.24 applies again, with a positive.
If the cable holding the elevator breaks, then a g , and Fe = 0; the man is weightless (in
the lift).
For instance, if the mass of the man is 100 kg, and a = 2 m/s2 as the elevator goes
upwards, then,
Fe m(10 2) 1200 N
As it slows down (motion still upwards) with an acceleration 1m / s 2 ,
Fe m(10 1) 900 N
FRICTIONAL FORCES
So far, we have considered motion without bringing into reckoning the frictional forces.
Frictionless motion is an idealised situation. In real life, there is a force opposing any kind
of motion, except in the case of a vacuum. Frictional forces always oppose motion.
However, frictional forces can also exist in cases where the body is static.

mg
Fig. 4.6

30

Consider the case where a block rests on a rough surface as shown in Fig. 4.6. If we try to
pull it to the right with a force, F , we notice that there is a kind of reluctance, that is, the
block refusing to move. This is because the force we are exerting is not enough to pull
the block free of the frictional force between it and the surface. However, we can increase
the force to a point where the block is just about to slip. The force is maximum at this
point. Thus, at this point, we define the coefficient of static friction. As soon as the block
starts moving, the force necessary to keep it moving with uniform velocity is less than that
needed to set it in motion. Then, we talk of dynamic friction.
At the point where the block is just about to slip, we define the coefficient of static friction
as,
F
s
N
Where N is the normal reaction the surface exerts on the block, as the block is not
breaking the surface, nor the surface pushing it off. Thus, the minimum force needed to set
the block in motion is F s N . In other words, we can write,
f s s N
as the frictional force being exerted by the surface on the block, which prevented it
from moving, until we supplied the maximum force the frictional force between the two
bodies could be.
We could therefore define the coefficient of static friction as the ratio of the maximum
static frictional force between the two bodies and the magnitude of the normal reaction.
Also, we could therefore define the coefficient of kinetic friction as the ratio of the
maximum kinetic frictional force between the two bodies and the magnitude of the normal
reaction.
As soon as the body starts moving,
fk k N
where k is the coefficient of kinetic friction.
The Inclined Plane
Fig. 5.6 depicts a plane inclined to the horizontal at an angle .
f mg cos

N mg cos

mg sin

mg cos

mg
Fig. 5.6

31

Suppose we increase the angle from zero, the force tending to pull the block down the
incline is mg sin . However, this force is opposed by the frictional force f . At the value
of s when the block is just about slipping, we define the coefficient of static friction s
and hence at this angle, the frictional force experienced by block is s mg cos s where the
normal reaction exerted by the surface on the block is N mg cos s . Then,
mg sin s s mg cos s
Hence,
s tan s
By measuring the angle of inclination that makes the block just about to slip, we can find
the coefficient of static friction between the surface and the block.
Once the body starts sliding down the plane, then we can define the coefficient of kinetic
friction,
k tan k
where k is the angle of inclination required to ensure that the block slides down the plane
with constant speed.
DYNAMICS OF UNIFORM CIRCULAR MOTION
We noted that in uniform circular motion, the centripetal acceleration is given as,
v2
a
2r
r
There is a corresponding force, called the centripetal force, which is given as,
v2
F ma m
m 2 r
r
This force, very much like centripetal acceleration, is directed towards the centre of the
circle. The fact that this force is directed at right angles to the velocity (which is always
along the circumference, indeed, the tangent to the curve) ensures that it does not change
the magnitude of the velocity.
If there is a force pointing into the centre of the circle, to keep the body in uniform circular
motion on the circle, there is the need for another force, called the centrifugal force, which
is equal in magnitude, but opposite in direction.
The centrifugal force is a fictitious force, in the sense that it appears because there is a
centripetal force. To know why this is so, consider a body tied to a string that is being
whirled in a horizontal circle. If the string should break, the body flies off tangential to the
point on the circle where this occurs. Thus, as the centripetal force disappears, the
centrifugal force also disappears. The centrifugal force is due to the inertia of the body,
because, according to Newtons first law, a body in uniform motion would like to stay in
that state unless an external force acts on it. The body would have loved to continue to
move straight, but for the force pushing it towards the centre of the circle. The reluctance
to leave a straight line creates the centrifugal force.

32

Banking
Let us consider the case where a car is moving round a bend of radius r with a velocity v .
2
v
Then, the centrifugal force it experiences is m . This force is balanced by the centripetal
r
force provided by the friction at the tyres. However, this (frictional) force is not large
enough, indeed, cannot be relied upon, when the speed of the car is large (notice the
square dependence of the centrifugal force on the speed.
If the road is banked, that is, it looks like Fig. 5.7,

N cos
900

N sin

mg
Fig. 5.7
N cos mg
v
N sin m
r
Therefore,
2
v
tan
rg
Thus,
v rg tan

33

mv 2 / r

PRODUCTS OF VECTORS
Here, we take a detour to learn how to find the products of vectors. This is very important,
as we shall get to know how a scalar or a vector could be obtained from the product of
vectors.
Dot or Scalar Product
The dot or scalar or inner product of two vectors A and B is defined as,
A B AB cos
where A is the magnitude of A and B is the magnitude of B. We could write
A Ax i Ay j Az k

x.1
x.2

where i , j and k are unit vectors in the x , y and z directions respectively.


Similarly,
B Bx i B y j Bz k
x.3
Then, the dot product of A and B is,
( Ax i Ay j Az k ) ( B x i B y j B z k )
= Ax B x i i Ax B y i j Ax B z i k
+ Ay B x j i Ay B y j j Ay B z j k
+ Az B x k i Az B y k j Az B z k k
= Ax B x Ay B y Az B z

x.4

It is simply the sum of the product of the respective components.


We note, therefore, that the dot or scalar product gives a scalar.
From equation (x.1), it is clear that the dot product of two vectors in the same direction,
= 0, is the product of the magnitudes. The dot product for two vectors that are
perpendicular is zero, since 900 .
From equation 5.2, we can also find the angle between two vectors,
AB
cos
AB
Example 1
Find the dot product of the vectors (1, -1, 2) and (3, 2, 1). Exercise.
Example 2
Find the angle between the vectors given in Example 1.

A 12 (1) 2 2 2 = 6 , B 32 2 2 12 = 14
([1 3] [1 2] [2 1])
cos
= 0.3273
6 14
Hence, tan 1 0.3273 = 18.120.

34

x.5

Cross or Vector Product


Given two vectors A and B , we define the cross product A B as follows,
i
j
k
A B = Ax Ay Az = i ( Ay B z Az B y ) j( Az B x Ax B z ) k ( Ax B y Ay B x )
Bx B y Bz
It is a vector whose magnitude is given as,
| A B | AB sin
where is the angle between the two vectors. The direction is given by that of the
cockscrew rule (Fig. 5.1). Note that B A (A B) .

A B
B

A
B A
Fig. 5.1
Note that the cross product of two vectors is in a plane perpendicular to the plane
containing the two vectors. In other words, it is perpendicular to each vector.
Example
Find the cross product of A (1, 3, 1) and B (2,1, 2) .
Solution
A B i ( Ay B z Az B y ) j( Az B x Ax B z ) k ( Ax B y Ay B x )
= i([3 2] [1 1]) j([1 2] [1 2]) k ([1 1] [3 2])
= 5i 5k
Alternatively,
i

A B 1 3 1 = i([3 2] [1 1]) j([1 2] [2 1]) k ([1 1] [2 3])


2 1 2

= 5i 5k

35

CHAPTER FIVE
WORK AND ENERGY
Work Done by a Constant Force
The work done by a force F over a displacement x is the dot product of the two vectors,
5.1
W Fx
Thus, the work done by a force is a scalar.
We can also write equation 5.1 as,
W Fx cos
where is the angle between F and x .

5.2

Yet again, we can write 5.2 as,


5.3
W F ( x cos )
We conclude that we could indeed, define the work done as the magnitude of the force
times the component of the displacement in the direction of the force.
On the other hand, we can also write,
5.4
W ( F cos ) x
Or, the work done is the magnitude of the force times the component of the force in the
direction of the displacement.
Note that in Fig. 5.1,
z y
cos
F x

y x cos
z F cos

F
Fig. 5.1
The unit of work is the Joule (J), which is the product of the Newton and the metre, the
units, respectively of force and displacement, J Nm .
Example
Find the work done by the force (-1, 2, -2) N when it pushes a box from (0, 1, 2) to (2, 1, 3) m.
Solution
The force is (1, 2, 2) .
The displacement is (2,1, 3) (1, 2, 2) (2 1,1 2,3 2) = (3, 1, 1) .
Thus, the work done is,
(1, 2, 2) (3, 1, 1) = 3 2 2 3 J

36

Work Done by a Variable Force


Consider a variable force. We have to get the force done by the force over an infinitesimal
displacement,
5.5
dW Fdx
Then, the total work done is the integral of equation 5.5.
W

x0

W dW Fdx

5.6

Special Cases
1.
Gravitational Force
In the case of gravity, the force is constant, and always directed towards the centre of the
earth, and is the weight of the body,
FG mg j
5.7
where j is the unit vector in the vertical direction, directed upwards.
The work done by this force over a displacement hj (h being positive means the
displacement is opposite the direction of the force) is,
WG mgh
5.8
Thus, gravity does negative work when the motion is upwards.
When motion is downwards, say hj , the work done is
WG mgh
5.9
Positive work is done by gravity when a body is dropping in the field of the earth.
2.
Elastic force
The force in this case is governed by Hookes law,
5.10
F kx
since it is a restoring force. In other words, if you stretch a spring, the spring tries to return
to its original length.
The work done by the elastic body is therefore,
x
x
1
5.11
We Fdx kxdx = kx2
0
0
2
It does not matter whether it is stretched or compressed. The force experienced by the
body is always against the force acting on it.
Energy
The energy possessed by a body is its ability to do work. It follows that the unit of energy
is also the unit of work, the Joule.
Work done on a body is stored as energy.
Kinetic Energy
The kinetic energy of a body is the work it does on account of its motion.
The work done by a body of mass m in moving through a distance dx is,

37

dW Fdx madx m

dv
dx
dx m dv mvdv
dt
dt

5.12

Hence,

W
0

1
dW W mvdv mv 2
v0
2
v

=
v0

1 2 1 2
mv mv0
2
2

5.13

This is the change in kinetic energy of the body when its velocity changes from v 0 to v .
We see, therefore, that the work done on a body is equal to the change in the kinetic
energy of the body. This is the work-energy theorem.
Work Done by a Conservative Forces, Potential Energy
A conservative force is such that the work done in a cycle is zero. It also means that the
work done does not depend on the path taken. The mechanical energy of such a system is
conserved. Thus, we can write,
M.E. = K.E + P.E = constant
5.14
If the change in kinetic energy is written as K and the change in potential energy is U ,
then
5.15
K U 0
Or
5.16
K U
But from the work-energy theorem,
5.17
W K
So,
5.18
W U
Or more precisely,
dW Fdx dU
We conclude that,
dU
F
dx
where U is the potential energy (function).
The student can then show that,

5.19
5.20

W W0 Fdx U ( x0 ) U ( x)

5.21

x0

where U is a function of position only. Note that if x x0 , then, W 0 , again buttressing


the fact that when the force is conservative, the work done in a cycle is zero. Indeed, only
in this case does a potential energy function exist.
We can then write equation 5.14 as,
K .E U ( x) E

5.22

While U (x) depends on the nature of the problem, the kinetic energy is always
Hence, we can rewrite equation 5.22 as,
1 2
mv U ( x) E
2

1 2
mv .
2
5.23

38

From equation (5.21),


x

U ( x) Fdx U ( x0 )

5.24

x0

We can arbitrarily choose the value of U ( x0 ) , and therefore, without loss of generality,
we can choose our x 0 conveniently, and set U ( x0 ) = 0. For instance, when dealing with a
body falling in this class, we can set the floor equal to x 0 and then we can take the
potential energy of a body as zero if it is on the floor. On the other hand, we could also set
the centre of the earth as the zero of the potential energy. In any case, we can always write
equation (5.24) as,
x

U ( x) Fdx

5.25

x0

This is minus sign multiplied by the work done by the conservative force.
Let us consider again the two examples above.
1.
Gravitational Force
In the case of gravity, the force is constant, and always directed towards the centre of the
earth, and is the weight of the body,
FG mg j
where j is the unit vector in the vertical direction, directed upwards.
The work done by this force over a displacement hj (h being positive means the
displacement is opposite the direction of the force) is,
WG mgh
Thus, gravity does negative work when the motion is upwards.
h

U (h) mgdy = mg (h h0 )

5.26

h0

We bear in mind that going up means h0 h . Thus, the potential energy of the body
increases as it goes up. If we set the earths surface equal to zero potential energy, then
U (h) mgh
5.27
When a body rises from height h 0 to height h ,
U mg (h h0 )
The kinetic energy reduces
1
1
K mv 2 mv02
2
2
K U 0
1 2 1 2
mv mv0 mgh mgh0 0
2
2
Hence,
1 2
1
mv mgh mv02 mgh0
2
2

39

5.28
5.29

5.30

5.31

When motion is downwards, say hj , the work done is


WG mgh
h

U ( x) mgdy = mg (h h0 ) mg (h0 h)
h0

5.32

Positive work is done by gravity when a body is dropping in the field of the earth, but this
constitutes a fall in the potential energy of the body, since h h0 . This is equal to mgh
if we take the surface of the earth as h0 0 . Yet again,
when a body falls from height h to height h 0 ,
U mg (h0 h)
5.33
The kinetic energy increases
1
1
5.34
K mv02 mv 2
2
2
K U 0
1 2 1 2
5.35
mv0 mv mgh0 mgh 0
2
2
Again,
1 2
1
5.36
mv mgh mv02 mgh0
2
2
Thus, this relation holds whether the body is going up or falling in the gravitational field
of the earth.
v 2 v02 2 g (h h0 )
Or
5.37
If h 0 is lower than h , then body is rising up, and you get,
v 2 v 02 2 gh

5.38
However, if we set h 0 as the top of a building say, and h as a point lower than h 0 , then,
v 2 v02 2 g (h h0 ) v02 2 g (h0 h) v02 2 gh
Recall these two formulas (5.38 and 5.39) from O Level Physics?

5.39

2.
Elastic force
The force in this case is governed by Hookes law,
F kx
since it is a restoring force. In other words, if you stretch a spring, the spring tries to return
to its original length.
The work done by the elastic body is therefore,
x
x
1
We Fdx kxdx = kx2
0
0
2
The potential energy,
1
U kx2
2
Hence, we can write,
1 2 1 2
mv kx E
2
2
where E is a constant.

40

5.40

5.41

Therefore, considering two points,


1 2 1 2 1 2 1 2
mv kx mv0 kx0
2
2
2
2

5.42

Examples
1.
A ball drops from the top of a 10 m tall building. Find its velocity halfway down.
Solution
DIY
2.

A catapult is stretched 0.2 cm. If the stiffness constant is 250 N/m, find the
velocity of the stone of mass 20g when released.

41

CHAPTER SIX
CENTRE OF MASS, CENTRE OF GRAVITY
Let a set of masses m i be arranged along the x -axis. Then, we can define the centre of
mass of this linear arrangement as
n

m x
i 1
n

cm =

m
i 1

Thus, for the body shown below, a set of masses arranged on a weightless beam, the centre
of mass is calculated as follows:
5 cm
2 cm
0 1.5 cm
7 cm
The centre of mass is
2kg (5cm) 4kg (2cm) 1kg 1.5cm 3kg 7cm
(2 4 1 3)kg
10 8 1.5 21
4 .5
=
cm = 0.45 cm
cm =
10
10
Note that if the origin had been taken to be at the extreme left of the beam, then the centre
of mass would have been
2kg (5cm) 4kg (2cm) 1kg 1.5cm 3kg 7cm
(2 4 1 3)kg
10 8 1.5 21
40.5
=
cm = 4.05 cm
cm =
10
10
In three dimensions (in Cartesian coordinates), the formula for centre of mass is at
n

m r
i 1
n

i i

m
i 1

where ri is the position vector of mass m i from the origin, or


ri xi i y i j z i k
Thus, cm is at

n
n
n

i
m
x

j
m
x

k
mi xi

i i i i
n
i 1
i 1

mi i1

i 1

Of course, if in only x-y plane, it reduces to

42

cm =

n
n

i
m
x

j
i i mi xi
n
i 1

mi i1

i 1

Example
Find the centre of mass of the rigid body shown below (Take the acceleration to be
uniform, and equal to 10 m/s2.)
7
6

1 kg

2 kg

5
4

1.5 kg

3 kg

3.5 kg

1
0
0

2 2 3 2 1.5 4 3.5 4 1 6
2 3 1.5 3.5 1
4 6 6 14 6
=
= 3.2727
11
2 5 3 2 1.5 3 3.5 1 1 5
cy
2 3 1.5 3.5 1
10 6 4.5 3.5 5
=
= 2.6364
11
cx

Centre of Gravity
The centre of gravity of a body is defined in a similar way, but with the weight replacing
the mass.
Hence, centre of gravity is at
n
n
1 n

i
w
x

j
w
x

k
wi xi

i i i i
n
i 1
i 1

wi i1
i 1

In the case where the acceleration due to gravity is constant throughout the composite
body, then
n
n
1 n

i
m
gx

j
m
gx

k
mi gxi

i i i i
n
i 1
i 1

mi g i1
i 1

43

n
n
n

i
m
x

j
m
x

k
mi xi

i i i i
n
i 1
i 1

mi i1

i 1

That is, the centre of gravity is the same as the centre of mass if the acceleration due to
gravity is constant throughout the extent of the composite body.

44

CHAPTER SEVEN
CONSERVATION OF LINEAR MOMENTUM AND COLLISIONS
When the net force acting on a body is constant, then the time rate of change of linear
momentum is zero, that is,
d
p0
dt
meaning that p = constant. That is, the velocity is constant. This is Newtons first law.
When two bodies are involved, then if there is no net external force on the system,
d
d
p (m1 v1 m2 v 2 ) 0
dt
dt
The velocities at a later time are denoted by prime ( ' )
Or, m1 v 1 m2 v 2 m1 v 1' m2 v '2
Rearranging, we see that m1 ( v 1' v 1 ) m2 ( v '2 v 2 ) . But these are the changes in the
momentum of each of the bodies. We conclude that the momentum lost by one is gained
by the other.
Example
A gun-bullet system has zero total momentum before the gun is fired, but as soon as it is
fired, the momentum gained by the bullet in moving forward is lost by the gun which
moves backwards, as the total momentum must again be zero.
COLLISIONS (1-Dimension)
There are two types of collision: Elastic and Inelastic
Elastic collision
In elastic collision, both the kinetic energy and the linear momentum are conserved.
Thus,
1
1
1
1
m1v12 m2 v22 m1v'12 m2 v' 22
2
2
2
2
and
m1v1 m2 v 2 m1v1' m2 v 2'
We can rewrite the energy equation as
m1 (v12 v'12 ) m2 (v' 22 v 22 )
(*)
and the linear momentum equation
m1 (v1 v'1 ) m2 (v' 2 v 2 )
(**)
Dividing (*) by (**) , gives
v1 v1' v 2' v 2
or
v1 v 2 (v 2' v1' )
The student should be able to show that

45

v1'

(m1 m2 )
2m 2
v1
v2
m1 m2
m1 m2

v 2'

(2m1 )
(m m1 )
v1 2
v2
m1 m2
m1 m2

Inelastic Collisions
When the collision between two bodies is not elastic, the total linear momentum of the
system is still constant, but the total kinetic energy is reduced. Indeed there is a period of
compression and a period of restitution. The latter period is that in which the bodies
recover their shapes after the initial compression.
u1

u2

Thus, m1u1 m2 u 2 m1v1 m2 v2


1
1
1
1
m1u12 m2 u 22 m1v12 m2 v22
2
2
2
2
The coefficient of restitution is
speed of separation of particles
=e
speed of approach of particles

0 e 1 , with the limits corresponding to completely inelastic collision (e = 0, in which


case the bodies stick together after collision) and completely elastic collision (e = 1).
In the figure above, u1 u 2 (vectorially), or there will be no collision.
The speed of approach is u1 u 2 and the speed of separation is v2 v1 , because v 2 must
be greater than v1 or they would not separate.
e

v 2 v1
0
u1 u 2

Examples
1.
Find the value of e if m1 travels towards the right at 4m/s and m 2 is at rest, and
after collision, m1 travels to the right at 2 m/s and m 2 travels to the right at 4 m/s.
Solution
6m/s

0m/ s

2m/s

4m/ s

46

42 1
=
60 3

2. Find the value of e if m1 travels towards the right at 4m/s and m 2 travels at 2m/s
towards the left, and after collision, m1 travels to the left at 1 m/s and m 2 travels to
the right at 3 m/s.

4m/s

2m/ s

1m/s

3m / s

3 1 2
=
42 3

Example
Two particles A and B of mass 0.2 kg and 0.5 kg respectively are moving towards each
other along the same straight line on a smooth horizontal table. Particle A has speed 12
m/s and particle B has speed 2 m/s. Given that the coefficient of restitution between the
particles is 0.2, find
(a) The speed of A and B after the impact.
(b) The total mechanical energy of the system before and after impact.
Solution
Before impact
12 m/s
m1 0.2kg

2m/ s
m2 0.5kg

After collision
v1 m/s

v2 m / s

From the law of conservation of momentum,


m1u1 m2 u 2 m1v1 m2 v2
(0.2)(12 ) (0.5)( 2) (0.2)v1 (0.5)v2
2v1 5v2 14
The coefficient of restitution is
speed of separation of particles
=e
speed of approach of particles

47

(*)

v2 v1 1

14
2
Therefore, v 2 v1 7
(**)
From (*) and (**), v1 3 and v2 4 , meaning that each particle reversed after the
collision.

The kinetic energy before collision is


1
m1u12 m2 u 22 0.5 [0.2 12 2 0.5 2 2 ] 15.4 J
2

The kinetic energy after collision is


1
m1v12 m2 v22 0.5 [0.2 32 0.5 4 2 ] 4.9 J
2

Hence, the loss in kinetic energy is 10.5 J.


The lost kinetic energy is converted, possibly, into heat, sound or some other form of
energy.
Exercise
A small smooth sphere A of mass 2 kg is travelling along a straight line on a smooth
horizontal plane with speed 6 m/s when it collides with a small smooth sphere B of mass 3
kg moving along the same straight line in the same direction with speed 2 m/s. After the
collision, A continues to move in the same direction with speed 3 m/s.
(a) Find the speed of B after the collision.
(b) Find the coefficient of restitution between A and B.
(c) State any assumption you made in your calculations.

48

CHAPTER EIGHT
NEWTONS LAW OF UNIVERSAL GRAVITATION
Newtons law of universal gravitation states that the force of attraction between two
masses m1 and m 2 is directly proportional to the product of their masses and inversely
proportional to the square of the distance between them.
mm
8.1
F G 12 2
r
G is the Universal gravitational constant = 6.67 10 11 Nm 2 kg 2 .
Example
Find the gravitational force between the electron and the proton in the hydrogen atom
( me 9.1 10 31 kg , m p 1.67 1027 kg ; take r = 10 15 m )

Solution

F G

me m p

6.67 10 11

r2
= 1.0137 10 37 N

9.1 10 31 1.67 10 27
(10 15 ) 2

For a body close to the surface of the earth, of mass m, we can write
mM
F G 2
re
where M is the mass of the earth, and re is the radius of the earth.
mM
F G 2 = mg e
re
GM
It follows therefore, that 2 g e .
re

8.2

8.3

The mass of the earth is 6 10 24 kg , and its radius, 6400 km = 6.4 10 6 m


Hence,
6.67 10 11 6.24 10 24
= 10.16 m / s 2
ge
6 2
(6.4 10 )
A more precise value of g is 9.8 m / s 2 . We made some approximations in our
calculations.
Exercise
Find the mass of a man on a mysterious planet that has three times the mass of the earth if
the radius of the planet is twice that of the earth.
Variation of Acceleration due to Gravity
We know that for a body of mass m on the surface of the earth,

49

GmM e
mg e
8.4
re2
where g e is the gravitational acceleration on the surface of the earth. What if the body is
actually at a point distant r from the centre of the earth. Then,
GmM e
8.5
F'
mg '
r2
But GmM e is constant, and from equation 8.4, this is mg e re2 . Hence,
F

re2
F ' mg e 2 mg '
r
Therefore,
r2
g ' e2 g e
r
This is an inverse square law.

8.6

8.7

Now, outside the earth, r re h


2

re
1
h
h

g'
ge
g e 1 g e 1 2 ... g e
2

h
re
re h
re

1
r
e

If h re , we can neglect higher order terms in the expansion, and write,

h
g ' 1 2 g e
re

8.8

8.9

Inside the earth, only the part of the earth below the body contribute to the gravitational
pull. Let the body be at a distance a from the centre of the earth. Then, the effective mass
of the earth involved in the gravitational pull is,
M ' (4 / 3) a 3
But M e (4 / 3) re3 ,

M ' a3
a3
, or M ' 3 M e

M e re3
re

8.10

Therefore,

F''

aGmM e
GmM ' Gm a 3
mg ' '
2 3 Me =
2
re3
a
a
re

GM e
ge
re2
Therefore,
a
g'' ge
re
This is a linear function of a .

8.11

But

8.12

Sketch the functions in (8.7) and (8.12).

50

Gravitational Field, Gravitational Potential and Escape Velocity


The gravitational force experienced by a body is given as,
GmM
F
r2
where m is in the field of the mass M .

8.13

The gravitational field of mass M is the gravitational force exerted on a unit mass, i.e.,
GM
8.14
g 2
r
The work done by the gravitational field in moving a mass m from ra to rb in the field of
M is,
rb
rb dr
1 1
8.15
W Fdr GmM 2 GMm
ra
ra r
rb ra
The gravitational potential energy of M is,
r
GMm
1 1
8.16
U Fdr GMm

r
r
and is the work done in bringing mass m from infinity to a point at radial distance r in the
gravitational field of M.
The gravitational potential is the work done in bringing a unit mass from infinity to a
point at radial distance r in the gravitational field of M,
GM
8.17
V
r
For a body launched from the earth to overcome the gravitational pull of the earth, it must
have a kinetic energy equal to the gravitational potential energy, that is,
GmM e 1 2
mv
8.18
re
2
or
2GM e
v
= 2 g e re
8.19
re
GM e
or GM e g e re2
2
re
This is the escape velocity of the earth.

since g e

Parking Orbits
A satellite is said to be in parking orbit when it has the same angular frequency as the
body it orbits. That is, it goes round the object once as the object completes its revolution
on its axis. Thus, for a satellite to be in parking orbit of the earth, it must complete its
revolution round the earth in 24 hours, meaning that it remains over the same spot on the
earth at all times.

51

GmM e mv 2

8.20
r
r2
where r is the radial distance of the satellite from the centre of the earth.

Hence,
v2

But
v

GM e g e re2

r
r

8.21

2 r
T

8.22

Hence,
4 2 r 2 g e re2

8.23
r
T2
4 2 r
8.24
T2
g e re2
For a satellite to be in parking orbit,
T 24 60 60s
Putting in the values of g e , re , the student can confirm that the distance from the centre of
v2

the earth for a parking orbit is 4.24 103 km. This is about 3.6 103 km above the surface
of the earth.

52

CHAPTER NINE
RIGID BODY STATICS
Moment of a Force or Torque
The torque or moment of a force about a reference point is the vector given by
= rF
where r is the position vector of the point of application of the force relative to the
reference point.

O
Fig 9.1

Note that the angle between the two vectors is the angle between their directions. Thus, we
could redraw Fig. 9.1 as

F
r
O
Fig. 9.2

The magnitude of the moment is | | = | r F || r || F | sin , where is the angle


between the two vectors.
Notice that the moment of a force tends to make a body the force is acting upon to rotate
about the reference point O. It is important, therefore, to refer to the reference point. You
would recall that you were taught at the secondary school level, about clockwise and
counter-clockwise (or anticlockwise) moment. This is because, at least at this level, the
moment makes the body rotate either clockwise (as in the case above) or counterclockwise
about the reference point.
Let us take a look at opening a door. Do you push the door close to the hinges? Do you
push the narrow edge of the door parallel to the door through the line through the hinges?
No. It is easier to push the door at the farthest end (from the hinges) and perpendicular to
the length of the door. Thus, if the door is of length r and the force you apply is F, in case,
the moment of the force is rF sin 90 rF . Of course, if 0 , the moment of the force is
zero.
The moment of a force is a vector, the direction of which is given by that of the righthanded cockscrew. It follows that if your hand should move from r to F contained on the
plane of the paper, sweeping through the angle (clockwise in this case), the vector
moment of the force points away from you, into the paper. The vector for anticlockwise
moment would point towards you.

53

Equilibrium of a Rigid Body


Earlier, we saw what it takes for a point mass to be in equilibrium the net force on it
must be zero. For a rigid body (an extended body), we require, additionally, that the net
moment on the body be zero. This is summed up in two laws:
(i)
The net force acting on the body in any direction is zero. You have been taught
the equivalent of this force: sum of upward forces is equal to the sum of
downward forces. This is just only one part of it. Indeed, there cannot be a net
force in any direction, otherwise, the body would move in that particular
direction.
(ii)
The net moment about any point on the body must be zero. This you saw as:
sum of clockwise moment = sum of anticlockwise moment. This applies to any
point at all, as if there is any point at which the net moment is not zero, that
point would rotate about the reference point.
Example 1:
As an example, consider a uniform beam of negligible weight resting on a fulcrum as
shown in Fig. 9.3. It is of length 1 m, and is resting on a knife edge placed at the 40 cm
mark. There is a 50 g mass hanging from the 80 cm mark. Clearly, the beam cannot be
balanced. There must be a force on the other side of the 50 g mass to ensure balance. Let
us decide to hand a 200 g mass on that side. Where exactly should we put it?
R
0.2m

0.4m

0.4m
1m

Let us place the 40 g mass at a point x m from the left end of the beam.
xm

R
0.4m

0.4m
200g

1m

0.2m

50g

Fig. 9.4
Condition 1: The sum of upward forces equals the sum of downward forces. The reaction
at the support equals the weight of the beam. In this case, indeed, we have not bothered
with the weight of the beam as it is negligible (otherwise, we would have had its weight
pointing down from the center of gravity, same as center of mass in this case).
Condition 2: Net Moment about the fulcrum is zero, that is, sum of clockwise moments
minus sum of anticlockwise moments is zero:
9.1
0.05kg g 0.4m [0.2kg g (0.4 x)m] = 0
Hence,

54

0.05
0.25
0.2
x 0.4 0.25 = 0.15 m

0.4 x

Example 2:
A metre rule of mass 30 g is supported at the 40 cm and the 85 cm marks. Two masses 45
g and 52 g are hung from the 42 cm and the 90 cm marks respectively. Find the reaction at
the supports.

R1

R2
0.85 m

0.4 m
.42 m

45 g

m = 30 g

52 g

0.9 m
cm
Fig. 9.5
(iii)
(iv)

The sum of upward forces = sum of downward forces:


R1 R2 (.045 0.03 0.052 ) 10 1.27 N
Taking moments about R1 :
R2 0.45 0.045 g 0.02 0.03 g 0.1 0.52 g 0.4

9.2
9.3

From equation 9.3, we can obtain R 2 . Putting this in equation 9.2 gives R1 .
Example 3:
A man of mass m is climbing a uniform ladder of weight W and length L m resting on a
rough floor and a smooth wall, as shown in Fig. 9.6. Find the reaction at the wall. How far
up the ladder can he go without the ladder slipping if the coefficient of static friction is
s ?

F1
L

F2

W
F2 x

55

F2 y

Fig. 9.6

Solution
The forces acting on the ladder are already shown. F2 is the force the ladder exerts on the
floor, and is a resultant of what the reaction would have been without friction F1 y , and the
frictional force F2 x . There is no friction at the wall. Hence, only the normal reaction F1
needs be reckoned with.
Condition 1: Sum of upward forces = sum of downward forces
F2 y W mg
9.4
F1 F2 x
9.5
Let the man be x m up along the ladder. Then, if the angle the ladder makes with the floor
is , then taking moments about the foot of the ladder,
L
9.6
F1 H W cos mgx cos
2
If the center of gravity of the beam had been one-third of the way up the ladder, we would
have replaced L / 2 by L / 3 . Indeed, we replace L / 2 by L / p , if the center of gravity is
one-pth of the distance up the ladder.

From equation 9.4, we can calculate F2 y , since W and mg are known. From equation 9.6,
we can get find F1 and hence, F2 x (they are equal) in terms of x. But we recall that this
force can be written as F2 x s F2 y . We can then get x since each term on the right is
now known. As an example, consider the following:
A man of mass 55 kg is climbing a uniform ladder of weight 70 kg and length 10 m resting
on a rough floor and a smooth wall. If the ladder touches the wall at a height of 7 m, how
far up the ladder can the man go without the ladder slipping, given that the coefficient of
static friction between the ladder and the rough floor is 0.3?
F2 y W mg = 70 10 55 10 1250N

9.7

F1 F2 x
9.8
Let the man be x m up along the ladder. Then, if the angle the ladder makes with the floor
is , then taking moments about the foot of the ladder,
L
F1 H W cos mgx cos
2
10
7
7

F1 7 70 10 cos sin 1 55 10 x cos sin 1


2
10
10

7 F1 3500 .7141 550 x .7141


2499.35 392.755x
F1 357 .05 56 .108 x = F2 x = s F2 y , where s is the coefficient of static friction

between the ladder and the rough floor.


F1 357 .05 56 .108 x 0.3 1250 375

56

Hence, x 0.3199 m.

CHAPTER TEN
RIGID BODY DYNAMICS
A rigid body is indeed such that the relative position of the constituent masses remain
constant. Thus, if we consider Fig. 10.1., then
mn

r1

m1

rn

r2
m2

r3

m3

Fig. 10.1
Let the body rotate about O with angular frequency . Then, the total kinetic energy of
the rigid body is
n
1
1
1
1
1
2
2
2
2
2
KE m1v1 m1v1 m1v1 ... mn v n mi vi
2
2
2
2
i 1 2
n
n
1
1
mi ( i ri ) 2 mi ( ri ) 2
i 1 2
i 1 2
(since the body is rigid, each constituent rotates at the angular frequency )
Hence,
n
1
1
2
KE 2 mi ri I 2
10.1
2
2
i 1
n

where I mi ri is called the moment of inertia of the rigid body.


2

i 1

You would observe that the expression for the kinetic energy looks like that for kinetic
1
energy mv 2 if we make the identification v and m I . Thus, moment of inertia
2
plays the role of the equivalent of mass for the rotational motion of a rigid body.

Moment of Inertia of a Continuous Body


We have seen that the moment of inertia of a rigid body with discrete components is
n

I mi ri . We shall now see the situation where the body is of continuous distribution.
2

i 1

57

For one, the summation becomes an integral, I r 2 dm , and the integral evaluated as the
problem demands.

Examples
1.

A thin rod of length rotating about an axis perpendicular to one of its ends

dm
x

Fig. 10.2
Clearly, the elemental mass dm contributes an elemental moment of inertia x 2 dm .
The total moment of inertia is then,
L

I one end x 2 dm x 2 Adx


since dm A dx , the product of the density and the volume of the element. Note that the
volume is A dx , where A is the cross-sectional area and dx is the (elemental) length of the
elemental mass. and A are constant. Hence, we can write,
I one end A

L
0

x3
x dx A
3

L3
3

10.2
But AL M , the mass of the cylinder, or M / AL . Putting this in the expression for
I one end ,
I one end

M L3 1
A ML2
AL 3 3

10.3

Let us now see what happens when the bar rotates around its center about an axis
perpendicular to the bar.

dm
x

L/2

L/2
Fig. 10.3

58

I middle 2 A

L/2
0

x3
x dx 2 A
3

L/2

L3
12

But AL M , the mass of the cylinder, or M / AL . Putting this in the expression for
I middle ,
I middle

M L3
1
A ML2
AL 12 12

10.4

Parallel Axis Theorem


You would notice that we could have got equation 10.3 from equation 10.2 as follows:
1
1
3 1 2 1
L
I one end I middle M ML2 ML2
ML ML2
12
4
12
3
2
This is indeed true in the general case of an axis through a point at a distance h from the
axis through the center of mass,
I I cm Mh 2
2

Assignment
Find the moment of inertia of uniform rod through an axis parallel to its length, passing
through a point one-third to one of its ends.
A body could have both rotational and translational motion. Such a body would then have
both translational kinetic energy and rotational kinetic energy, so we could write the total
kinetic energy as,
1
1
10.5
KE Mv2 I 2
2
2
Thus if a uniform solid cylinder of radius R and mass M should roll down an inclined
plane from height h from rest, the gravitational potential energy Mgh is fully converted to
1
the translational kinetic energy Mv2 and rotational kinetic energy
2
1 2 11

I MR 2 2 at the bottom of the incline, where v and have their values at the
2
22

bottom of the incline. The moment of inertia of a solid cylinder about an axis through its
1
center, parallel to its length is I MR 2 .
2
The total kinetic energy is then
1
1
1
1
2
1
3
KE Mv2 MR2 2 Mv2 Mv2 Mv2 Mv2 Mv2
2
4
2
4
4
4
4
3
Mgh Mv 2 ,
4
or

59

4
gh
3
In the absence of rotation, all the potential energy is converted to translational kinetic
energy,
1
Mgh Mv 2
2
or
v 2gh
v

4
gh
3
This is because some of the potential energy has been converted to rotational motion.
2 gh

Assignment
A solid cylinder of mass 3 kg and radius 0.2 m is propelled up an inclined plane with an
initial velocity of 25m/s. How far up the incline can the cylinder travel?

ANGULAR MOMENTUM AND TORQUE


The angular momentum of a body is the moment of its momentum. Thus, if the
momentum of the body is p , then, its angular momentum about a reference point is,
10.6
L rp
It is therefore, a vector the direction of which is given by the right-handed cockscrew rule.
Just as the moment of a force gives the rotational effect of the force, so does the angular
momentum give the rotational effect of a momentum.
We can write the magnitude of the angular momentum as
L rp sin mvr sin
where is the angle between the two vectors.

10.7

If the vectors are perpendicular, L = r p = mvr, and zero if the vectors are parallel. Makes
sense, right? Just as it was in the case of the moment of a force, the rotating effect is zero
if the two vectors are in the same direction.
Let us now take the time-derivative of the angular momentum.
d
dp dr
L r
p
dt
dt dt
= r F v p r F v mv r F mv v = r F =
Remember we said the moment of a force about a reference point is the torque.

10.8

Thus, we conclude that the torque a body in angular motion is the time rate of change of
its angular momentum, that is,

60

dL
dt

10.9

Note that differentiating a cross product means that we keep the order of appearance of the
vectors involved, since cross product is not commutative, that is, r p p r .
dL
dL
, what happens if the torque is zero? Then,
0 , and the angular
dt
dt
momentum is constant. Does this equation look like the one you saw earlier? That is,
dp
0 . Newtons first law! In just the same way, in angular motion, a body remains in
dt
angular motion with constant angular momentum until a net external torque acts on it.
dp
Can you also see that equation 10.9 is like F
, Newtons second law? You can then
dt
see again that torque plays in angular motion the equivalent of force in translational
motion.

Since =

In uniform circular motion, 0 , hence, there is no torque on a body in uniform circular


motion. This is why the angular momentum remains L rp mvr mr 2 , because is
constant in uniform circular motion.
In circular motion, r and p are perpendicular (remember?), because the velocity is along
the circumference. Hence, we can write | L | L rp mvr mr 2 . Then, the torque is,
d
d
(mr 2 ) mr 2
I (r is constant, but is not)
dt
dt

Example
A flywheel of radius 1.2 m and moment of inertia 12.5 kgm2 starts rotating from rest and
uniformly attains an angular speed of 10 rad/s2 in 5 seconds. Find the force applied along
the rim of the flywheel.

Solution
The torque on the flywheel is I rF , where r is the radius of the wheel.
The angular acceleration is given by,
0 10

= 5 rad/s2
t
2
Then,
I 12.5 2
= 20.8333 N
F

r
1.2
Notice that the moment of inertia of a thin rod rotating about an axis through the middle is
less than that of the same rod through an axis at the end of the rod for instance. This is

61

why a rod rotating about an end would be more effective or destructive than the one
rotating through the middle if they are both rotating at the same angular speed - I .
The more the moment of inertia, the more the torque causing the motion, or the amount of
destruction the rod can cause.

62

DPHS 101 TUTORIAL QUESTIONS


1. Vector A has a magnitude of 8.0 at an angle of 60 0 from the x -axis, vector B has a
magnitude of 6.0 at an angle of 30 0 from the x -axis. Determine the vector sum
C = A + B by resolution into components.
2. Find the difference D = A B for the vectors given in Problem 1.
3. Find the vector V = 2A B for A and B given in Problem 1.
4. A boat is travelling in a river with a current of 3 km/hr. The boat is capable of
travelling at 10.0 km/hr in still water.
(a)
How long will it take the boat to travel 7 km upstream?
(b)
How long will it take to travel 7 km downstream?
5. A boat capable of making 9.0 km/hr in still water is travelling upstream in a river
flowing at 4 km/h. An object lost overboard is not missed for 30 minutes.
(a)
If the boat is then turned around, how long will it take to overtake the
floating lost object?
(b)
What total distance will the boat travel relative to the shore from the
point of turnaround to the point of overtaking the object?
6. Two cars approach each other on the highway. Car A moves north at 90 km/hr, car
B moves south at 70 km/hr.
(a)
What is the velocity of car A as seen from car B?
(b)
What is the velocity of car B as seen from car A?
(c)
What are their velocities relative to car C, which is travelling north at
100 km/hr?
7. A boat capable of making 9 km/hr in still water is used to cross a river flowing at a
speed of 4 km/hr
(a)
At what angle must the boat be directed so that its motion will be
straight across the river?
(b)
What is its resultant speed relative to the shore?
8. A monkey on a cliff throws a coconut horizontally from a height of 17 m with a
speed of 2.1 m/s. If the ground below the cliff is level, how far from the base of the
cliff does the coconut strike the ground?
9. An object on a horizontal plane starts at the origin and moves with velocity v x = 10
cm/s, v y = 4 cm/s.
(a)
(b)
(c)

How far is it from the origin after 5s?


In what direction is it from the origin?
In what direction is it moving?
10. (a)
How far will a stone travel over level ground if it is thrown upward at
an angle of 300 with respect to the horizontal and with a speed of 12 m/s?
Assume the stone is launched at a height of 2.0 m above the ground.
(b)
What is the maximum horizontal range that could be achieved with the
same initial speed?
11. What is the linear speed of the earth in its orbit round the sun? Give your answers
in metres per second (consider the earths orbit to be a circle of radius 1.5 1011 m).
12. A medivac helicopter travels 78 km due south of its base to pick up a patient. The
helicopter then travels 93 km due north to a hospital. The entire trip takes 1.22 hr.
(a)
What is the average velocity?
(b)
What is the average speed?

63

13. Two automobiles travel in opposite directions from the same starting point. If the
speed of one is twice the speed of the other and they are 200 km apart at the end of
1 hour, what is the speed of each car?
14. A motorcyclist moving with an initial velocity of 8 m/s undergoes a constant
acceleration for 3 s, at which time his velocity is 17 m/s.
(a)
What is the acceleration?
(b)
How far does he travel during that 3 s interval?
15. A small parachute dropped from a 30m-high cliff falls with a constant velocity of
1.2 m/s. Twenty seconds after the parachute is dropped, a stone is dropped from
the cliff. Will the stone catch up with the parachute before it reaches the ground?
16. A boy drops a stone each second from a very high window.
(a)
How far has the first stone gone when the second one is dropped?
(b)
Does the distance between the first and the second sinker remain
constant? Explain your answer.
17. A loose bolt falls from a high-flying helicopter that is rising at a constant 8.76 m/s.
How far is the bolt below the helicopter 3.05 seconds later?
18. A ball dropped from the top of a tower reaches a velocity v f just before it reaches
the ground. An automobile travelling towards the tower with constant speed v f
just reaches the tower when the ball strikes the ground. Show that at the instant the
ball was released, the distance of the car from the tower was twice the height of the
tower.
19. From the position-versus-time graph of Fig. 1, find the average velocity from t = 0
s to (a) t = 10 s, (b) t = 20 s, (c) t = 40s.
100

Metres

50
0
0

10

20

30

40

50

-50
-100
Time (s)

Fig. 1
20. In Fig. 1, what is the instantaneous velocity at (a) t = 5s, (b) t = 15s, (c) 25 s, (d) 35
s.
21. Draw the instantaneous velocity-versus-time graph corresponding to Fig. 1.
22. Draw the acceleration-versus-time graph corresponding to Fig. 2.

64

v0

2T0

T0

4T0

3T0

5T0

time

6T0

v0

Fig. 2
23. When does the acceleration corresponding to Fig. 2 have (a) a maximum value, (b)
a minimum value, (c) a value of zero?
24. A car starts from rest at t = 0 and accelerates as shown in Fig. 3. Find its velocity at
(a) t = 10s, (b) t = 30 s, (c) Draw the velocity-versus-time graph.
a ( ms 2 )

4
2

t (s)
2
4
Fig. 3
25. Plot the position-versus-time graph corresponding to Fig. 4 (assume the object
starts from x = 0).
v (ms 1 )

10

t (s)
10

Fig. 4
26. A 75-kg man stands in an elevator. What force does he exert on the floor of the
elevator under the following conditions?
(a)
The elevator is stationary.
(b)
The elevator accelerates upwards at 2m/s2.
(c)
The elevator rises with constant velocity at 4 m/s.
(d)
While going up, the elevator accelerates downwards at 1.5 m/s2
(e)
The elevator goes down with constant velocity of 7m/s.

65

27. A fish of mass m is suspended by a string as shown in Fig. 5. The string is


fastened securely at point C but will pull loose from the wall at A when the string
tension exceeds 22 N. What is the maximum mass of the fish that can be supported
by the string?

Fig. 5
28. Suppose that the weight W2 in Fig. 6 is 400 N. What must be the values of the
weights W1 and W3 ?

W1

W3

W2

Fig. 6
29. A 10kg block is placed on a frictionless inclined plane and connected to a 5kg
block as shown in Fig. 7.
(a)
What would the angle have to be for the blocks to remain
motionless?
(b)
What would be the acceleration of the blocks if = 370.

66

Fig. 7
30. For Fig. 8, describe what will happen and what the spring scale S will read if (a)
m1 = m 2 = 1 kg, (b) m1 1.2 kg, m 2 = 1 kg (ignore the mass of the scale.

Fig. 8
31. The period of a stone swung in a horizontal circle on a 2m cord is 1s.
(a)
What is its angular velocity in rad/s?
(b)
What is its linear speed in m/s?
(c)
What is its radial acceleration in m/s2.
32. The earth is 1.5 1011 m from the sun and has a period of about 365 days. Assume
the earths orbit to be circular and determine the magnitude and direction of its
radial acceleration in km/s2.
33. Jupiters moon Europa has an average orbital radius of 6.67 108 m and a period of
85.2 hrs. Calculate the magnitude of
(a)
the tangential velocity
(b)
the angular velocity
(c)
the centripetal acceleration
34. A stunt pilot in an airplane diving vertically downward at a speed of 220 km/hr
turns vertically upward by following an approximately semi-circular path with a
radius of 180 m (Fig. 9).
(a)
How many g s does the pilot experience due to his motion alone?
(b)
By what factor does the pilots weight appear to increase at the bottom
of the dive?

67

Fig. 9
35.
36. What is the centripetal acceleration of an automobile driving at 40 km/hr on a
circular track of radius 20 m?
37. A race track curve has a radius of 100 m and is banked at an angle of 680. For what
speed was the curve designed?
38. A fly of mass 0.2 g sits 12 cm from the centre of a phonograph record revolving at
1
33 rpm.
3
(a)
What is the magnitude of the centripetal force on the fly?
(b)
What is the value of the coefficient of static friction between the fly and
the record.
39. A small ball is fastened to a string 24 cm long and suspended from a fixed point P
to make a conical pendulum, as shown in Fig. 10. The ball describes a horizontal
circle about a centre vertically under point P, and the string makes an angle of 150
with the vertical. Find the speed of the ball.

Fig. 10
40. What is the minimum force required to drag a carton of books of mass 40 kg across
the floor if the force is applied at an angle of 450 to the horizontal? Take the
coefficient of static friction as 0.6.
41. A bob of mass m is whirled in a circular path on the end of a string 1.0 m long. If
the string makes an angle of 200 with the vertical (Fig. 11), what is the tangential
speed of the ball?

68

Fig. 11
42. A 100-kg crate is pushed at constant speed up a frictionless 300 ramp (Fig. 12).
(a)
What is the horizontal force F required?
(b)
What force is exerted by the ramp on the crate?

Fig. 12
43. A trunk with a weight of 220 N rests on the floor. The coefficient of static friction
between the trunk and the floor is 0.41, while the coefficient of kinetic friction is
0.32.
(a)
What is the minimum magnitude for a horizontal force with which a
person must push on the trunk to start it moving.
(b)
Once moving, what magnitude of horizontal force must the person
apply to keep the trunk moving with constant velocity?
(c)
If the person continues to push with the force used to start the motion,
what would be the acceleration of the trunk?
44. The coefficient of kinetic friction in Fig. 13 is 0.20. What is the acceleration of the
block if
(a)
it is sliding down the slope, and
(b)
it has been given an upward shove and is still sliding up the slope.

69

Fig. 13
45. The two blocks (with m = 16 kg and M = 88 kg) shown in Fig. 14 are not attached.
The coefficient of static friction between the blocks is s = 0.38, but the surface
beneath M is frictionless. What is the minimum magnitude of the horizontal force
F required to hold m against M ?

Fig. 14
46. As shown in Fig. 15, a 1.34-kg ball is connected by means of two massless strings
to a vertical, rotating rod. The strings are tied to the rod, are taut, and form two sides of
an equilateral triangle. The tension in the upper string is 35 N.
(a)
(b)
(c)
(d)

Draw the free-body diagram for the ball.


What is the tension in the lower string?
What is the net force on the ball at the instant shown in the figure?
What is the speed of the ball.

Fig. 15

70

KINEMATICS
1.

A box sits on a horizontal wooden board. The coefficient of static friction between
the box and the board is 0.5. You grab one end of the board and lift it up, keeping
the other end of the board on the ground. What is the angle between the board and
the horizontal direction when the box begins to slide down the board?

Solution:
The critical angle is determined by the condition:
From this equation we can find an angle 26 0
2.

An archer shoots an arrow with a velocity of 30 m/s at an angle of 20 degrees with


respect to the horizontal. An assistant standing on the level ground 30 m
downrange from the launch point throws an apple straight up with the minimum
initial speed necessary to meet the path of the arrow. What is the initial speed of
the apple and at what time after the arrow is launched should the apple be thrown
so that the arrow hits the apple?

Solution:
The motion of the arrow is a projectile motion. Then the motion of the arrow along
horizontal direction is a motion with constant velocity:
Where the initial position is 0 and

. Then

Motion along vertical axis is a motion with constant acceleration, then

Where

, initial position is 0, and

. Then

Then we need to find the time when the arrow will be exactly above the assistant.
At this moment of time x(t) = 30. Then we can find the time:
Then we can find the height of the arrow at this moment of time:

The assistant should throw the apple with minimal velocity so it will reach point
5.4 m and at this height the velocity should be 0. From this condition we can find
the minimal velocity:
The time of the motion of the apple to this point is

Then the apple should be thrown after

71

3.

An 8 kg block is at rest on a horizontal floor. If you push horizontally on the 8 kg


block with a force of 20 N, it just starts to move.
(a) What is the coefficient of static friction?
(b) A 10.0 kg block is stacked on top of the 8 kg block. What is the magnitude F of
the force, acting horizontally on the 8 kg block as before, that is required to make
the two blocks start to move?

Solution:
The magnitude of horizontal force should be equal to the magnitude of the
maximal static friction force, which is equal to the product of the coefficient of
static friction and the normal force (gravitation force in the present problem).
(a) The gravitation force is mg=8*9.8 = 78.4 N. Then the coefficient of static
friction is
(b) Now we know the coefficient of static friction and we know the normal force:
18*9.8 = 176.4 N. Then we can find the magnitude of force F:

4.

You are driving along the street at the speed limit (35mph) and 50 meters before
reaching a traffic light you notice it becoming yellow. You accelerate to make the
traffic light within the 3 seconds it takes for it to turn red. What is your speed as
you cross the intersection? Assume that the acceleration is constant and that there
is no air resistance.

Solution:
This is the motion with constant acceleration. If the acceleration of the car is a then
we can write the expression for traveled distance:

We know the initial velocity


. We
also know that after 3 seconds the car travels distance 50 meters. Then we can find
acceleration

Now we know acceleration, then we can find the final velocity:

5.

The car drives straight off the edge of a cliff that is 57 m high. The investigator at
the scene of the accident notes that the point of impact is 130 m from the base of
the cliff. How fast was the car traveling when it went over the cliff?

72

Solution:
Motion along axis x (horizontal axis) is the motion with constant velocity. So we
can write down the dependence of x-coordinate as a function of time:
Where v is the initial velocity (the initial velocity has only x-component, its
direction is along axis x).
We know the final x-coordinate of the car it is 130 m. But we do not know the
traveled time and the initial velocity.
We can find the traveled time from the motion along axis y. This is the motion
with constant acceleration (free fall acceleration). We know the height of the cliff
this is the traveled distance in y-direction. We know that initial velocity (in ydirection) is 0. Then we can write the following equation:

From this equation we can find the traveled time:

Then from the motion along axis x we have:


From this equation we can find initial velocity:

6.

A tortoise and a hare are in a road race to defend the honor of their breed. The
tortoise crawls the entire 1000 meters at a speed of 0.2 m/s. The rabbit runs the
first 200 meters at 2 m/s, stops to take a nap for 1.3 hours, and awakens to finish
the last 800 meters with an average speed of 3 m/s. Who wins the race and by how
much time?

Solution:
At first let us calculate the travel time of tortoise. We know the speed and the
distance, so we can easily find the time:

Now let us calculate the traveled time of rabbit. The traveled time consists of three
parts:
1. He runs the first 200 m at 2 m/s. The time of this motion is

2. Then he takes a nap for 1.3 hours:

73

3. Then he runs the last 800 m with speed 3 m/s. The time of this motion is

Then the total travel time is

Since t p t r then tortoise wins the race by 47 s.


7.

A "moving sidewalk" in a busy airport terminal moves 1 m/s and is 200 m long. A
passenger steps onto one end and walks, in the same direction as the sidewalk is
moving, at a rate of 2.0 m/s relative to the moving sidewalk. How much time does
it take the passenger to reach the opposite end of the walkway?

Solution:
The speed of the passenger relative to the ground is 2m/s+1m/s =3 m/s (since the
he is walking in the same direction as the direction of the motion of the sidewalk.
Then the passenger reaches the end of the sidewalk after

8.

(a) If a particle's position is given by x 6 12t 4t 2 (where t is in seconds, and x


is in meters), what is its velocity at t = 1s?
(b) What is its speed at t=1s?
(c) Is there ever an instant when the velocity is 0? If so, give the time.

Solution:
The velocity is the derivative of x(t) with respect to time. Then

(a) at t = 1 s we get:
What we calculate here is the x-component of velocity. The negative sign means
that the direction of velocity is opposite to the direction of axis x.
(b) the speed is the magnitude of velocity. Then at t = 1 s the speed is 4 m/s
(c) to find time at which the velocity is 0 we just need to solve an equation:
From this equation we find time:

At this moment of time the velocity is 0.

74

8.

The captain of a plane wishes to proceed due west. The cruising speed of the plane
is 245 m/s relative to the air. A weather report indicates that a 38-m/s wind is
blowing from the south to the north. In what direction, measured due west, should
the pilot head the plane relative to the air?

Solution:
This is problem is on relative motion. The velocity of the plane is the vector sum of
the velocity of the wind (air) and the relative velocity of the plane (relative to the
air) as shown in the figure.
We know that the direction of the final velocity of the plane is from east to west (as
shown in the figure).
Then from the triangle base shown in the figure we can find angle . This angle
characterizes the direction of the relative velocity of the plane.

Then

9.

The highest barrier that a projectile can clear is 14 m when the projectile is
launched at an angle of 30.0 degrees above the horizontal. What is the projectile's
launch speed?

Solution:
The maximum height of the projectile is given by the equation:

where 300 is the launch angle. Then we can find the initial velocity:

75

10.

Snow is falling vertically at a constant speed of 3 m/s. At what angle from the
vertical do the snowflakes appear to be falling as viewed by the driver of a car
traveling on a straight, level road with a speed of 60 km/h?

Solution:
At first we need to have the same units, so we need to convert the speed of the car
in m/s:

Then we need to find the direction of the velocity of the snowflakes relative to the
car:

Then from the triangle shown in the picture we can find angle :

Then

11.

A missile is launched directly upward into the air at an initial velocity of 80 m/s. It
is moving with constant velocity until it reaches 1000m, when the engine fails.
(a) How long does it take it to reach 1000m?
(b) How high does the missile go?
(c) How long does it take for it to fall back to the earth?
(d) How long does it stay in the air?
(e) How fast is it going when it hits the ground?

Solution:

76

(a) Since initially the motion is with constant velocity, we can easily find the time
of the motion of the missile till it reaches the height 1000 m. The time is given by
the expression:

After this point there is free fall motion there is only one force acting on the
object (gravitational force) this force provides free fall acceleration 9.8 m / s 2 .
The initial velocity is 80 m/s pointing upward. The acceleration is pointing
downward. The initial height of the missile is 1000 m. Then the equations which
describe this motion are the following:

(b) To find the maximum height of the missile we can use the last equation. The
velocity at the maximum height is 0. Then

(c) To find the time when the missile hits the ground we need to use the first
equation:

When the missile hits the ground h=0. Then

From this equation we can find time: 24.6 s.


(d) Then we can find the time when the missile is in the air: it is the sum of the
time when it reaches 1000 m and the time when it hits the ground:

(e) To find the speed of the missile when it hits the ground we need to use the last
equation:
When the missile hits the ground h = 0. Then

v 802 2 * 9.8 *1000 = 161 m / s

77

Here we need to add the minus sign, which illustrate the fact that the direction of
velocity is downward.
12.

A skier is accelerating down a 30 degree hill at 3 m / s 2 .


(a) What is the vertical component of her acceleration?
(b) How long will it take her to reach the bottom of the hill, assuming she starts
from rest and accelerates uniformly, if the elevation is 300 m?

Solution:
(a) If we assume that the direction of the vertical axis is downward then the vertical
component is positive and is equal to

(b) This is the motion with constant acceleration then the equation which describe
this motion is the following:

The traveled distance (L) is related to the elevation (h) by the equation:

Now we know acceleration and the traveled distance then we can find the traveled
time:

13.

A rocket is fired vertically upwards with initial velocity 80 m/s at the ground level.
Its engines then fire and it is accelerated at 4 m / s 2 until it reaches an altitude of
1000 m. At that point the engines fail and the rocket goes into free-fall. Disregard
air resistance.
(a) How long was the rocket above the ground?
(b) What is the maximum altitude?
(c) What is the velocity just before it collides with the ground?

Solution:
The first part of the motion is the motion with constant acceleration at
. The
initial velocity for this motion is 80 m/s. Then we can write the equation, which
describe the dependence of height of the rocket on time:

From this equation we can find the time when the rocket reach the height 1000 m =
h:

78

The solution of this equation is 10 s. So after 10 seconds the engine fails. The
velocity at this moment of time is

After this moment of time we have free fall motion there is only one force acting
on the object (it is gravitational force) this force provide free fall acceleration.
The initial velocity is 120 m/s pointing upward. The acceleration is
pointing downward. The initial height of the rocket is 1000 m. Then the
equations which describe this motion are the following:

To find the maximum height of the rocket we can use the last equation. The
velocity at the maximum height is 0. Then

This is the answer to part (b).

To find the time when the rocket hits the ground we need to use the first equation:

When the rocket hits the ground h=0. Then

From this equation we can find time: 31 s.


Then we can find the time when the rocket is in the air: it is the sum of the time
when it reaches 1000 m and the time when it hits the ground:
This is the answer to part (a).

To find the speed of the rocket when it hits the ground we need to use the last
equation:
When the missile hits the ground h=0. Then

This is the answer to part (c).

79

14.

A rock is dropped from rest into a well. The sound of the splash is heard 4 s after
the rock is released from rest.
(a) How far below to top of the well is the surface of the water? (the speed of
sound in air at ambient temperature is 336m/s).
(b) If the travel time for the sound is neglected, what % error is introduced when
the depth of the well is calculated?

Solution:
(a) If h is the depth of the well then to find the time of the rock's fall we need to use
the following equation:

From this equation we can find time:

After the rock hits the water the sound of splash will propagate the distance h with
h
speed 336 m/s. After time t s
, the sound reaches the ground. Then the total
336
time taken is

This time is equal to 4 s. Then

From this equation we can find h:

(b) If we neglect the sound velocity then t = 4 s and we can find the height from the
equation:

If we compare this result with the result from part (a) then we can find an error:

15.

A dart is thrown horizontally with an initial speed of 10 m/s toward point P, the
bull's-eye on a dartboard. It hits at point Q on the rim, vertically below P, 0.2 s

80

later. What is the distance PQ? How far away from the dartboard is the dart
released?
Solution:
The initial velocity is horizontal; the magnitude is 10 m/s. Then the equation of
motion along horizontal axis has the following form:
After 0.2 seconds the dart hits the target. From this condition and from the above
equation we can find the distance between the point where the dart is released and
the board:

The equations which describe the motion along the vertical axis are the following:

If we substitute into the first equation the traveled time then we can find the
distance between points P and Q:
The minus sign means that point Q is below point P.
16.

A swimmer dived off a cliff with a running horizontal leap. What must his
minimum speed be just as he leaves the top of the cliff so that he will miss the
ledge at the bottom which is 2 m wide and 9 m below the top of the cliff?

Solution:
This is the projectile motion with horizontal initial velocity. In this case the motion
in horizontal direction is described by the following equation:
where v is the initial velocity.
The motion along vertical axis y is described by the following equations (ycomponent of initial velocity is 0):

We know that the final point of the motion is the end of the ledge. The coordinates
of the final point are: x=2 m and y = -9m.
We can substitute these numbers in the above equations and obtain:

81

From the seconds equation we can find time:

Then we substitute this time in the first equation and obtain the initial velocity:

17.

A ferris wheel with radius 20 m which rotates counterclockwise, is just starting up.
At a given moment, a passenger on the rim of the wheel and passing through the
lowest point of his circular motion is moving at 3.00m/s and in gaining speed at a
rate of 0.5 m / s 2 . Find the magnitude of the passenger's acceleration at the instant.

Solution:
The acceleration at this point has two components: the tangential acceleration,
which is equal to a t = 0.5 m / s 2 , and centripetal acceleration, which is given by the
expression:

These components are orthogonal. Then the net acceleration is

18.

In an action film, the hero is supposed to throw a grenade from his car, which is
going at 90 km/h, at his enemy's car, which is going 110 km/h. The enemy's car is
15.8 m in front of the hero's when he lets go of the grenade. If the hero throws the
grenade so its initial velocity relative to him is at an angle of 45(degree) above the
horizontal, what should the magnitude of the initial velocity be? The cars are both
traveling in the same direction on a level road. Ignore air resistance.
Find the magnitude of the velocity both relative to the hero and relative to the
earth.

Solution:
At first we need to convert all the velocities into m/s:

We will describe the motion in the hero's reference frame. In this reference frame
the velocity of the enemy's car is
And the grenade is released with the launch angle 450. We introduce the magnitude
of initial velocity of grenade (relative to the hero's car) as v.

82

After the grenade is released it is moving according to the equations of projectile


motion. Along axis x there is motion with constant velocity:
Along axis y there is motion with constant acceleration:

In these equations at the initial moment of time the grenade has zero coordinates.
We can also write down the equation of enemy's car motion:
Now we need to write down the condition that the grenade hits the enemy's car:
At some moment of time t 0 the x coordinate of grenade should be equation to the
x-coordinate of enemy's car and at the same moment of time the y-coordinate of
grenade should be equal 0.
Then we have:

Then we just need to solve the system of two equations with two unknown
variables:

Then
And
Then

This is the velocity of the grenade relative to the hero.


We can also find the magnitude of velocity relative to the earth. The x-component
of this velocity is
The y-component of the velocity is
Then the magnitude is

This is the magnitude of the velocity of grenade relative to the earth.


19.

A flowerpot falls from a windowsill 25.0 m above the sidewalk.

83

(a) how fast is the flowerpot moving when it strikes the ground?
(b) how much time does a passerby on the sidewalk below have to move out of the
way before the flowerpot hits the ground?

Solution:
This is the free fall motion with constant acceleration. The equations which
describe this motion are the following:

where we introduced an axis y with upward direction. The origin of axis y is the
initial position of the flowerpot. The initial velocity of flowerpot is 0.
We know the final position of the flowerpot: the coordinate of the final point is -25
m.
(a) We substitute this coordinate in the third equation an obtain the velocity:

The negative sign means that the direction of velocity is downward. The magnitude
of velocity is 22 m/s.
(b) The traveled time can be found from the first equation in the above system. We
substitute y(t) = -25 and obtain time:

20.

A firefighter, a distance d from a burning building, directs a stream of water from a


fire hose at angle above the horizontal. If the initial speed of the stream is v , at
what height h does the water strike the building?

Solution:
This is projectile motion. The equation which describe the motion along horizontal
axis x is the following

Along axis y we have motion with constant acceleration:

In these equations at the initial moment of time the water has zero coordinate.

84

What do we know about the final point: we know only the x-coordinate of the final
point it is x = d. We can substitute this coordinate in the first equation (which
describe the motion along axis x):
From this equation we can find the traveled time:

Then we substitute this time into the equation which describe the y-coordinate of
the water:

This expression gives as the height where the water strikes the building.
21.

You are driving towards a traffic signal when it turns yellow. Your speed is 55
km/h, and your best deceleration has the magnitude of a 5.15 m / s 2 . Your best
reaction time before braking is t 0 0.75 s . To avoid having the front of your car
enter the intersection after the light turns red, should you break to a stop or
continue to move at 55 km/h if the distance to the intersection and the duration of
the yellow light are
(a) 40m and 2.8s, and
(b) 32m and 1.8s?

Solution:
The first step we need to convert the speed to m/s:

We have two possible types of motion:


1. motion with constant speed till the driver reaches the intersection. Time of this
motion is
2. motion with deceleration. The driver starts deceleration after t 0 0.75 s . During
this time the driver travels the distance:

The equations which describe these motion are the following:

85

where the initial position of the car has zero coordinate.


At the final point the velocity is 0. Then

During this time the position of the car is

Now we can analyze the problem:


(a) 40m and 2.8s. Then for motion 1 we have:

Since 2.6<2.8 then the driver can cross the intersection if he will move with
constant speed.
(b) 32m and 1.8s. In this case:

Since 2.1>1.8 the driver cannot cross the intersection.


Therefore in this part we need to consider case 2. But even in this case the distance
he will travel before he stops is 34.2 m, which is greater than 32 m. Therefore he
cannot stop before the intersection.
22.

The tips of the blades in a food blender are moving with a speed of 20 m/s in a
circle that has a radius of 0.06 m. How much time does it take for the blades to
make one revolution?

Solution:
The traveled distance of a tip is
where R = 0.06 m.
Then the time is equal to the ratio of the traveled time and the average velocity:

23.

While you are traveling in a car on a straight road at 90 km/hr, another car passes
you in the same direction; its speedometer reads 120km/hr. What is your velocity
relative to the other driver? What is the other car's velocity relative to you?

Solution:
The relative velocity is equal to the difference of velocities.
So if you need to find your relative velocity then you need to subtract from your
velocity the velocity of another car:

86

The minus sign here gives us the direction of relative velocity: the direction is
opposite to the directions of motions of the cars.
Similarly we can find:

The plus sign here gives us the direction of relative velocity: the direction is the
same as the directions of motions of the cars.
24.

A tortoise can run with a speed of 10.0 cm/s, and a hare can run exactly 10 times as
fast. In a race, they both start at the same time, but the hare stops to rest for 3.00
min. The tortoise wins by 10 cm.
(a) How long does the race take?
(b) What is the length of the race?

Solution:
(a) If L is the length of the race then the tortoise run this race the time:

When the tortoise crosses the finish line the hare is 20 cm = 0.2 m behind tortoise.
It means that during time t tortoise it traveled the distance L - 0.2. Since he stopped
for rest for 3 minutes =3*60 s=180 s and his speed is 10 vtortoise 1 m / s , then we
can write the equation:

Taking into account the first equation we have:


Then

(b) Then we can find the length of the race:

25.

Emily takes a trip, driving with a constant velocity of 90 km/h to the north except
for a 30 min rest stop. If Emily's average velocity is 75 km/h to the north, how long
does the trip take?

Solution:

87

The average velocity is the ratio of traveled distance and the traveled time:

If the traveled time is t, then Emily traveled time (t-0.5)h with the speed 90 km/h.
Then the traveled distance is

We substitute this relation in the first equation and obtain:


Then

Then we can find L (length of the trip):

26.

To qualify for the finals in a racing event, a racecar must achieve an average speed
of 250 km/h on a track with a total length of 2000 m. If a particular car covers the
first half of the track at an average speed of 230 km/h, what minimum average
speed must it have in the second half of the event to qualify?

Solution:
The average speed is the ratio of traveled distance and traveled time:

For the whole track we have: v av 250 km / h and L 2000 m 2 km. Then the
traveled time is

The car travels the first half of the track (1 km) with speed 230 km/h. The time of
this motion is

Since the total traveled time should be 0.008 h, then the second part of the track the
car should travel 0.008-0.0043 = 0.0037 h. Since the distance of this motion is 1
km, then the average speed is

27.

You are trying to cross a river that flows due south with a strong current. You start
out in your motorboat on the west bank, desiring to reach the east bank directly
across from your starting point. Which direction should head your motorboat?

88

Draw a picture of the river, the banks, and your motorboat, and include the relevant
velocity vectors. What information would you need in order to determine the actual
direction you need to head?
Solution:
To find the actual direction of the boat velocity we need to know the velocity of
the river and the speed of the boat relative to the river (relative velocity). Then the
actual velocity (this is the vector) is

The diagram is shown below.

28.

The acceleration of an object as a function of time is a(t ) 8t (m / s 2 ) . Determine


the
(a) velocity and
(b) the position of the object as a function of time if it is located at x = 2 m and has
a velocity of 3 m/s at time t = 0 s.

Solution:
(a) By definition, the velocity of the object is the integral of acceleration with
respect to time:

where v0 3 m / s is the initial velocity (at t = 0).


(b) The position can found as an integral of velocity with respect to time:

89

where x 0 2 m is the initial position (at t = 0) of the object.


29.

An automobile traveling 90 km/h overtakes a 1.5-km-long train traveling in the


same direction on a track parallel to the road. If the train's speed is 70 km/h,
(a) how long does it take the car to pass it, and
(b) how far will the car have traveled in the time?

Solution:
It is easier to solve this problem if we introduce the relative velocity of the car
(relative to the train). The velocity of the car relative to the train is (90-70) km/h =
20 km/h.
In the relative description the train is not moving and the car is moving with
constant speed 20 km/h.
(a) Then we can easily find the time the car needs to pass the train. It is

(b) To find the actual traveled distance of the car we just need to multiply the
traveled time (0.075 h) by the actual speed of the car:

30.

Two cannonballs, A and B, are fired from the ground with identical initial speeds,
but with launch angle of cannonball A larger than the launch angle of cannonball
B.
(a) Which cannonball reaches a higher elevation?
(b) Which cannonball stays longer in the air?
(c) Which cannonball travels farther?

Solution:
(a) The vertical component of the initial velocity of the projectile is proportional to
sine of the launch angle. It means that the vertical component of initial velocity of
cannonball A is larger than the vertical component of cannonball B. Then
cannonball A reaches a higher elevation.
(b) The time in the air depends only on the vertical component of initial velocity of
projectile. The larger the vertical component of the velocity the larger the time in
the air. Then the cannonball A will stay longer in the air.
(c) From the data of the problem we cannot tell which one will travel farther. The
horizontal length is proportional to sin(2 * launch angle)) . It has maximum at angle
450. Therefore we can have both possibilities:

90

cannonball A travels further than cannonball B for example, if the launch angle of
cannonball A is 450 and the launch angle of cannonball B is 300.
Cannonball B travels further than cannonball A for example, if the launch angle of
cannonball B is 450 and launch angler of cannonball A is 500.

31.

A man is driving at the speed 40 mph when he sees an obstacle at distance 300 ft
ahead of his position. The driver applies the brakes and decelerates at 10 ft / s 2 .
How long does it take him to stop the vehicle? How far from the obstacle will the
driver be when he finally stops?

Solution:
At first we need to convert all the variables into correct units (SI units):

Then we have motion with constant deceleration. Then


The car stops when v(t ) 0 , then
And
The traveled distance is

Then the distance between the car and the obstacle is

32.

A body moves 4 km towards East from a fixed point A and reaches point B. Then
it covers 5 km towards North and arrives at point C. Find the distance and
directions of the net displacement.

Solution:
We show two displacements (travelled paths) in the figure.

91

The net (resultant) displacement is the vector sum of two displacements: the first

displacement ( s1 ) and the second displacement ( s 2 ):

The easiest way to find the net displacement is to introduce coordinate system
(axes x and y) and then find the x and y components of the net vector
displacement. The x and y components of the displacement ( s1 ) and displacement

( s 2 ) are the following:

Then the x and y components of the net displacement is

Then the magnitude of the net displacement is

The direction of the net displacement is characterized by angle (shown in the


figure), which can be found from the known x and y components of the net
displacement:

92

33.

A baseball player hits a homerun, and the ball lands in the left field seats, which is
120 m away from the point at which the ball was hit. The ball lands with a velocity
of 20 m/s at an angle of 30 degrees below horizontal. Ignoring air resistance
(A) find the initial velocity and the angle above horizontal with which the ball
leaves the bat;
(B) find the height of the ball relatively to the ground.

Solution:
(A) Initial velocity
Without air resistance this is simple projectile motion. In the present problem we
do not know initial velocity: we do not know the magnitude of the velocity (speed)
and we do not know its direction.
There are two sets of equations, which describe the motion of the projectile (ball).
Set 1: motion along horizontal axis (axis x see figure). This is the motion with
constant velocity. There is only one equation, which describe this motion:
................................................(1)
Here x0 0 .
Since the motion along the axis x is the motion with constant velocity then the xcomponent of the velocity is constant. We know the velocity at the final point.
Then we can find the x-component of the velocity at the final point:
This x-component of the velocity is equal to the x-component of the initial
velocity:
...........(2)
We also know the x-coordinate of the final point (point B): it is 120 m. We
substitute this value in equation (1) and obtain
From this equation we can find the time of travel from point A to point B:

Now we need to analyze the second set of equations.


Set 2: motion along vertical axis (axis y see figure). This is the motion with
constant acceleration free fall motion. There are three equations, which describe
this motion. Only two equations are independent, but it is convenient to write all
three equations:
.............(3)
................................................(4)

93

Since the initial y-coordinate is zero, then


.............................................(5)

We know the y component of the final velocity

This is the y-component of the velocity at the moment of time t AB 6.9 s . We


substitute these values in equation (4) and obtain

From this equation we can find the y-component of the initial velocity:

Finally we know the x- and y-components of the initial velocity:

From these expressions we can find the magnitude of the initial velocity and the
direction (angle) of the initial velocity:

94

Now we know the initial velocity.


(B) Final height
We need to find the final height of the ball (the final y-coordinate). To find the
final height we can use equation (3). We just need to substitute the y-component of
the initial velocity and the traveled time in this equation:

95

DYNAMICS
34.

You are driving along an empty straight road at a constant speed u. At some point
you notice a tall wall at a distance D in front of you. Would it require a larger force
to (a) continue moving straight and decelerate to a full stop before the wall, or (b)
turn left or right to avoid the wall? (to make the calculation easier assume that the
turn is done at a constant speed along a circular path).

Solution:
(a) If the car continue moving straight then the acceleration should satisfy the
following equation:
where v f , the final velocity, is 0, since the car should stop just before the wall. v i ,
the initial velocity, is equal to u. Then

If the mass of the car is m, then the force required to stop the car is

(b) Now the car is turning left or right to avoid the wall. This is the uniform
circular motion with a radius D. The acceleration of this motion is

Then the force required to turn the car is

We can see that in the case (a) the force is 2 times smaller.
35.

In the film 2001: A Space Odyssey, a wheel-like space station achieves artificial
gravity by spinning around its axis. If the station had a size of 2 km, how fast
should it be spinning for the people inside to feel the same gravitational
acceleration as on earth?

Solution:
If the space station is rotating with angular velocity then the acceleration
(centripetal acceleration) at distance R=2 km =2000 m is given by the equation

This acceleration should be equal to the free fall acceleration. Then

and

96

36.

As part a of the drawing shows, two blocks are connected by a rope that passes
over a set of pulleys. The block 1 has a weight of 400 N, and the block 2 has a
weight of 600 N. The rope and the pulleys are massless and there is no friction.

(a) What is the acceleration of the lighter block?


(b) Suppose that the heavier block is removed, and a downward force of 600 N is
provided by someone pulling on the rope, as part b of the drawing shows. Find the
acceleration of the remaining block.
Solution:
(a) The acceleration of the lighter block is equal by magnitude to the acceleration
of the other block. If the tension of the rope is T, then the equations of motion for
block 1 and block 2 are the following:

Then
and

(b) In this case the tension of the rope is given: T = 800N. Then the second
Newton's law for block 1 becomes:

97

37.

A boy of mass 40 kg wishes to play on pivoted seesaw with his dog of mass 15 kg.
When the dog sits at 3 m from the pivot, where must the boy sit if the 6.5 m long
board is to be balanced horizontally?

Solution:
We have equilibrium. It means that the net torque should be equal to 0. If x boy is
the distance from the boy to the pivot, then the equilibrium condition becomes:

38.

How fast should the earth spin in order for a 150 lb human not to be able to walk
on the ground?

Solution:
The condition that the human cannot walk on the ground is that there is no contact
between human and the ground. It means that the normal force is 0.
Let us assume that the earth is rotating with angular velocity . Then the human
on the ground will have centripetal acceleration with the magnitude
Where R 6.4 *106 m is the radius of the earth.
This acceleration is provided by the gravitation force and the normal force, so that
Then
Then condition that the normal force is 0 (or negative) is
Then
and

39.

Part a of the drawing shows a block suspended from the pulley; the tension in the
rope is 80 N. Part b shows the same block being pulled up at a constant velocity.
What is the tension in the rope in part b?

98

Solution:
In part (a) the equilibrium condition can be written as:

In part (b) we have the motion with constant velocity. It means that the
acceleration is 0 and the net force is 0, which can be written as
From these two equations we can find

40.

A box is sliding up an incline that makes an angle of 20 degrees with respect to the
horizontal. The coefficient of kinetic friction between the box and the surface of
the incline is 0.2. The initial speed of the box at the bottom of the incline is 2 m/s.
How far does the box travel along the incline before coming to rest?

Solution:
The first part in the problem is to find an acceleration of the motion. The
acceleration is due to gravitation force and the friction force and has the following
form:

The second part is to write down the kinematic equations of motion. In this
problem we need to use the relation between the traveled distance and initial and
final (the final velocity is 0) velocities:
where s is the traveled distance. Then

99

41.

A block weighing 80 N rests on a plane inclined at 30 degrees to the horizontal.


The coefficients of static and kinetic friction are 0.2 and 0.1 respectively. What is
the minimum magnitude of the force F, parallel to the plane that will prevent the
block from slipping?

Solution:
The minimum force corresponds to the condition that the static friction force has
the maximum value, which is 0.2*normal force. To find the normal force and the
external force we need to write down the condition of equilibrium: the net force is
0. Then we rewrite this equation in terms of x and y-components (x axis is parallel
to the plane).
The x-component of the second Newton's law has the form:

The y-component:

Then since f s s n , we obtain

Then

42.

A rocket of mass 5 *104 kg is in flight. Its thrust is directed at an angle of 60


degrees above the horizontal and has a magnitude of 7 *105 N . Find the magnitude
and direction of the rockets acceleration. Give the direction as an angle above the
horizontal.

Solution:
There are two forces acting on the rocket: the first one is the gravitation force and
the second one is the trust force. The net force (the sum of these two forces) will
provide an acceleration of the rocket. It is easier to find the x- and y-components of
the net force. If axis x is horizontal axis and axis y is a vertical axis then

Then acceleration can be found from the second Newton's law:

100

The magnitude of the acceleration is

The direction is characterized by the angle:


43.

The speed of a bobsled is increasing because it has an acceleration of 3 m / s 2 . At a


given instant in time, the forces resisting the motion, including kinetic friction and
air resistance, total 500 N. The mass of the bobsled and its riders is 300 kg.
(a) What is the magnitude of the force propelling the bobsled forward?
(b) What is the magnitude of the net force that acts on the bobsled?

Solution:
The net force in the sum of two forces friction (and air resistance) force and the
force propelling the bobsled forward. There are also normal force and gravitational
force, but they cancel each other. Then

The net force provides an acceleration of the bobsled. Then

Then we can find the force F forward :

44.

A soccer ball of diameter 35 cm rolls without slipping at a linear speed of 2 m/s.


Though how many revolutions has the soccer ball turned as it moves a linear
distance of 20 m?

Solution:
When the ball makes one turn it travels the distance

Then if the ball travels L=20 m = 2000 cm then it makes

45.

Starting from rest, a skier slides 200 m down a 35 degrees slope. How much longer
does the run take if the coefficient of kinetic friction is 0.3 instead of 0?

101

Solution:
The acceleration of the skier on a slope is
With zero friction we have:
With friction 0.3 we have:

To find the traveled time we need to use the equation:

Then

and

46.

The gravitational force that the sun exerts on the moon is perpendicular to the force
that the earth exerts on the moon.
The masses are: Mass of Sun 2 * 10 30 kg , Mass of Earth: 6 * 10 24 kg , Mass of
Moon: 7 * 10 22 kg . The distance between the sun and moon is 1 *1011 m , and
between the moon and the earth is 4 *108 m .
Determine the magnitude of the net gravitational force on the moon.

Solution:
The magnitude of the gravitation force that the sun exerts on the moon is

The magnitude of the gravitation force that the earth exerts on the moon is

Since the directions of the forces are orthogonal then the magnitude of the net
force is

102

47.

A bicycle is moving at 10 m/s. What is the angular speed of its tires if their radius
is 30 cm?

Solution:
The linear speed is related to the angular speed by the equation:
where
,

48.

. Then

A tension of 6000 Newtons is experienced by the elevator cable of an elevator


moving upwards with an acceleration of 2 m / s 2 . What is the mass of the elevator?

Solution:
From the second Newton's law we have
Then

49.

Is it easier to move a heavy box that is sitting on the ground by (a) pulling the box
from a rope that makes an angle 300 with the surface or (b) by pushing the box
with a force that makes the same angle (but pointing downwards) with the surface?

Solution:
In both cases the horizontal components of the tension force in the rope will be the
same, but the vertical components will have opposite direction.
Then the normal force in the case (a) will be less than the normal force in case (b).
Then the friction force (which is proportional to the normal force) will be less in
case (a) then in case (b).
Then it is easier to pull the box (case (a)) then to push it (case (b)).
50.

Consider a wet roadway banked, where there is a coefficient of static friction of


0.40 and a coefficient of kinematic friction of 0.2 between the tyres and the
roadway. The radius of the curve is R = 80m.
(a) If the banking angle is 300, what is the maximum speed the automobile can
have before sliding up the banking?

103

(b) What is the minimum speed the automobile can have before sliding down the
banking?
Solution:
(a) The direction of the forces and acceleration are shown in the figure. We write
down the second Newton's law in terms of components:

X (horizontal axis)- components:

y (vertical axis)- components:

We also have the relation between f s and n:

Then we have:
and

Then from the first equation (x-component):

Then

(b) In this case we have opposite direction of the friction force.

104

Then x and y components of the second Newton's law become:

We have the same relation between f s and n:


Then
and

Then from the first equation (x-component):

Then

51.

A circus clown weighs 900 N. The coefficient of static friction between the clown's
feet and the ground is 0.4. He pulls vertically downward on a rope that passes
around three pulleys and is tied around his feet. What is the minimum pulling force
that the clown must exert to yank his feet out from under himself?

Solution:
There are the following forces acting on the clown:
Tension force (applied to the hand) direction upward
Gravitational force direction downward,
Normal force direction upward;
Friction force direction horizontal;
Tension force (applied to clown's legs) direction horizontal.

105

Then the minimum tension force corresponds to the condition that the friction
force is equal to 0.4*normal force.
In terms of x (horizontal) and y (vertical) components we have:

Then

52.

An athlete, who has a mass of 100 kg, can throw a 500 g ball with a speed of 10
m/s. The distance through which his hand moves as he accelerates the ball forward
from the rest until he releases it is 1.0 m. What constant force must the athlete
exert on the ball to throw it with this speed?

Solution:
If we know the acceleration then we can find the force. Therefore the first step
should be to find the acceleration of the ball. We need to use the following
equation of motion with constant acceleration
The initial velocity is 0, the final velocity is 10 m/s and the distance travelled is 1
m. Then we can find acceleration

Then the force is

where the mass of the ball is 500 g = 0.5 kg.


53.

A 10.0 kg block is towed up an incline at 300 with respect to the horizontal. The
rope is parallel to the incline and has a tension of 100 N. Assume that the block
starts from rest at the bottom of the hill, and neglect friction. How fast is the block
going after moving 40 m up the hill?

Solution:
To find the speed of the block we need to find the acceleration of the block. The
speed is related to acceleration and traveled distance by the following equation

106

where h = 40 m. The acceleration is given by the equation

Then

54.

A 0.01 kg object is moving in a plane. The x and y coordinates of the object are
given by x(t ) 2t 3 t 2 and y (t ) 4t 3 2t . Find the net force acting on the object
at t = 2 s.

Solution:
To find the net force we need to find acceleration of the object. The acceleration is
the second derivative of coordinate of the object. Then the x and y components of
acceleration are given by the following expressions

Then at t=2 s we have

Then the magnitude of acceleration is

Then the net force is

55.

An automobile weighing 3200 lbs. is on a road which rises 10 ft. for each 100 feet
of road. What force tends to move the car down the hill?

Solution:
All variables in the problem should be expressed in the same units (the same
system of units). It is better to use the SI units. In the SI system of units, the mass
should be measured in kg and the distance is in meters. Then:

The forces acting on the car are the following: gravitation force, normal force,
friction force, and the force pulling the force up along the hill. The friction force is
the rolling friction force, which is small. The normal force does not have a
component along the hill (the normal force is orthogonal to the incline). Then there

107

is only one force pulling the car down the hill. This is the gravitational force,
shown in the picture below. The magnitude of the gravitation force is

Then from the picture of the incline we can find the component of the gravitational
force along the inline. This component can be expressed in terms of the angle of
incline, :
To find the angle of incline we need to use the right triangle, shown in the picture
above:

Then

56.

A 20 meters long rope attached to the top and the bottom of a flag pole is pulled 2
meters away from the pole by a 100 Newton force acting at right angles to the pole
at its mid point. What is the tension on the segments of the rope on each side of the
100 Newton force?

Solution:
In problem we need to use the equilibrium condition for point A: the vector sum of
all forces is zero. There are three forces acting on point A: an external 100 N force
and two tension forces . The directions of the tension forces are along the ropes.
These forces are shown in the figure below.

108

The y component of the external force is zero and the y-components of the tension
forces cancel each other. Then the condition that the x-component of the sum of all
these forces is zero takes the form:
Then

The angle can be found from the right triangle (with lengths of 10 m and 2 m),
shown in the figure:

Then

56. A crane cable that is capable of withstanding 22,000 N is attached by a hook to a


2,000 kg block that is resting on the ground. The cable initially starts lifting the
block at the maximum acceleration that the cable can withstand for 4 sec. It then
continues to raise the block at constant velocity for further 2 sec. At this time the
block slips off the hook at the end of the cable.
Calculate:
(1) the tension in the cable when the block is moving at constant velocity;
(2) the maximum acceleration that the cable can withstand;
(3) the maximum height that the block reaches above ground.
Solution:

109

(1) When the block is moving at constant velocity the acceleration of the block is
0. Then from the second Newton's law we get that the net force on the block is 0:
There are two forces acting on the block: gravitation force, pointing upward, and
the tension in the cable. The forces have opposite directions, then the magnitude of
the forces should be the same (the vector sum of these two forces is zero):
Therefore the tension in the cable is 19600 N.
(2) The maximum acceleration should be found from the condition that the tension
in the cable has its maximum value (22000 N). There are two forces acting on the
block: gravitation force and the tension in the cable. Then the second Newton's law
takes the form:
Then we rewrite this vector equation in terms of y-components (see figure below):
Then

(3) In the part we need to use the kinematics equations. First we have motion with
constant acceleration (we know its value from part (2)). The equations, which
describe this motion, are the following:

110

The initial velocity is zero: v0 0 , the initial height is zero: y 0 0 . Then we have

The block travels

. Then at this moment

Then the block moves with constant velocity. This velocity is v2 4.8 m / s . The
block travels for 2 seconds. The corresponding distance is

Then the final height of the block is

57.

A body of mass 10g is set to rotate in a circular path by means of a string 200 cm
long. If it makes 3 complete revolutions in 2s, find the tension of the string.

Solution:
The first step is to convert all variables into SI units:
mass should be measures in kg: m = 10 g = 0.01 kg;
length should be measured in meters: L = 200 cm = 2 m.
Now we can solve the problem. The body moves in a circle with constant speed.
This is the motion with acceleration centripetal acceleration. The acceleration is
pointing towards the center of the circle. The magnitude of acceleration is

Here is the speed of the body (the speed is constant) and L = 2 m is the radius of
the circle, which is equal to the length of the string. Therefore, to find the
magnitude of the centripetal acceleration we need to know the speed of the body.
We know that the body makes 3 complete revolutions in 2 seconds. During one
revolution the body travels a distance of 2L , then during 3 revolutions it travels a
distance of 6L in t = 2 s. Then the speed of the body is

Now we can find the magnitude of centripetal acceleration:

111

The centripetal acceleration is provided by the tension in the string. Then from the
second Newton's law we obtain the tension in the string:
58.

A particle moves in a circle of radius 1 m. Its linear speed is given by v 4t ,


where t is in second and v in meter/second. Find the radial and tangential
acceleration at t 2s .

Solution:
The tangential acceleration is defined as the change of the speed (magnitude of the
velocity) of the particle. Therefore the tangential acceleration is

The radial acceleration is centripetal acceleration, which changes the direction of


velocity. The centripetal acceleration is determined by the speed (not the change of
the speed) and the radius of circular orbit:

The velocity at t 2s is v 4t 4 2 8 m / s , then

59.

The force required to stretch Hooke's Law spring varies from 0 N to 65 N as we


stretch the spring by moving one end 6.3 cm from its unstressed position. Find the
force constant of the spring. Answer in units of N/m.

Solution:
We understand from the formulation of the problem that the spring force is 65 N
when the spring is stretched by x 6.3 cm , which in SI units is
x 6.3 cm 0.063 m .
At the same time from the Hooke's law we know that the spring force is
where k is the force constant of the spring. We substitute the known values on the
above expression and obtain
Then

60.

A 1.5 kg mass is attached to the end of a 90 cm string. The system is whirled in a


horizontal circular path. The maximum tension that the string can withstand is 400

112

N. What is the maximum number of revolutions per minute allowed if the string is
not to break?
Solution:
In this problem we need to use the second Newton's law and the fact that this is a
motion with acceleration centripetal acceleration. There are two forces acting on
the mass: gravitational force and the tension force. The vector sum of these two
forces provides the acceleration of the mass.
..................................................(1)
The centripetal acceleration is pointing towards the center of circle (rotation in a
circle).
To characterize the direction of the string we introduce angle .

Now we can rewrite equation (1) in terms of x and y components:


x-component:
..................................................(2)
y-component:

113

Now we need to use the expression for centripetal acceleration in terms of the
angular velocity and the radius of circular orbit:
.......................................................(3)
The radius of circular orbit can be expressed in terms of the length of the string and
the angle :
We substitute this expression in equations (2) and (3) and obtain

We can divide both sides of this equation by

and obtain

We know the tension has the maximum value: 400 N. Then we can find the
angular velocity:

If we know the angular velocity then we can find the period of rotation:

This is the time of one revolution. To find the number of revolutions per one
minute (60 seconds) we need to divide 60 seconds by the time of one revolution:

61.

A 20 kg child and a 30 kg child sit at opposite ends of a 4 m seesaw that is pivoted


at its center. Where should another 20 kg child sit in order to balance the seesaw?

Solution:
In this problem we need to use only one equilibrium condition: no rotation of the
seesaw (the seesaw is balanced). The second equilibrium condition (no
translational motion) needs to be used only if we need to find the normal force at
the pivot.
We can introduce any point as the point of possible rotation. We consider point O
the center of the seesaw as the point of possible rotation. There is no rotation
about point O, which means that the net torque about point O is zero.
First, we need to show all forces acting on the seesaw:
1. Gravitational force (more exactly it is normal force, which is equal to the
gravitational force) m1 g 20 9.8 196 N on the 20 kg child. Application point of
the force is one end of the seesaw (see figure). The distance to point O is d1 2 m .

114

2. Gravitational force (more exactly it is normal force, which is equal to the


gravitational force) m2 g 30 9.8 294 N on the 30 kg child. Application point
of the force is the other end of the seesaw (see figure). The distance to point O is
d2 .
3. We place another 20 kg child on the seesaw. We characterize the position of the
child by the distance to point O (this is an unknown distance): x . The
corresponding force is the gravitational force m3 g 20 9.8 196 N on the child.

There is no rotation about point O. Then the net torque about point O is zero:

We substitute the known values and obtain

From this equation we can find the position of the 20 kg child:

115

CONSERVATION LAWS
62.

A 10-kg box moving at 5 m/s on a horizontal, frictionless surface runs into a light
spring of force constant 100 N/cm. Use the work-energy theorem to find the
maximum compression of the spring.

Solution:
We need to write down the energy conservation in the present problem:
The initial energy is just the kinetic energy of the box:

The final energy is the elastic energy of the spring:

Then
and

63.

A 2000 kg truck is traveling east through an intersection at 2 m/s when it is hit


simultaneously from the side and the rear. One car is a 1000 kg compact traveling
north at 5 m/s. The other car is a 1500 kg midsize traveling east at 10 m/s. The
three vehicles become entangled and slide at one body. What are their speeds and
direction just after the collision?

Solution:
This is the example of perfectly inelastic collision: after the collision all parts of
the system move as a whole. To find the final velocity we need to use the
conservation of the net momentum.
The initial net momentum has three contributions:
- momentum of 2000 kg truck;
- momentum of 1000 kg compact;
- momentum of 1500 kg midsize.
We show these vehicles and the directions of the corresponding velocities (before
the collision).

116

Then the (initial) net momentum is

In the final state all vehicles move with the same velocity. Then the final
momentum is
Then momentum conservation takes the form

117

We need to rewrite this vector equation in terms of components. We introduce axis


x - east direction, and axis y north direction. Then the x- component of the above
equation is
Then

The y- component of the above vector equation is


Then

Now we know x and y components of the final velocity. Then the magnitude of
the final velocity is

The direction of the final velocity is characterized by the angle between the
direction of velocity and axis x (east):

64.

A cat stuck up a tree and has 500 J gravitational potential energy. It then falls. Find
the kinetic energy of the cat just before it is caught by the owner. Find the kinetic
energy of the cat after it is caught by the owner. Find the wasted energy (after cat
is caught by the owner).
Disregard air resistance.

Solution:
As the cat falls there is a transformation of potential energy into kinetic energy.
Just before the cat is caught by the owner the kinetic energy is exactly equal to the
initial potential energy = 500 J.
After the cat is caught by the owner, its velocity is 0. It means that the kinetic
energy is equal to 0 (no motion).

65.

The wasted energy is defined as an initial energy minus the final energy. The initial
energy is 500 J, the final energy is 0. Then the wasted energy is 500 J.
A metal surface is illuminated one by one by photons of energy 2 eV and 4 eV
respectively. The work function of the metal is 0.5 eV. What is the ratio of the
maximum speeds of electrons emitted in two cases?

Solution:

118

In this problem we need to use the energy conservation. The work function of the
metal can be considered as the potential energy of electron. Then the conservation
law in the present problem takes the form: the energy of the photon is equal to the
mechanical energy of electron:
The kinetic energy is

Then

and

Then the ratio of the speeds of the electrons in two cases is

66.

A small body of mass m is attached to a light thread which passes through a hole at
the centre of a smooth table. The body is set into rotation in a circle of radius r1
with a speed of v1 . The thread is then pulled down shortening the radius of the path
to r2 . What will the new linear speed

and the new angular speed 2 be?

Solution:
In the present problem we have conservation of the angular momentum. The initial
angular momentum is
The final angular momentum is

Since

then

From this equation we can find the final speed of the body

119

The angular speed can be expressed through the linear speed and the radius of
circle:

Then

67.

Ball A, with a mass of 2 kg, moves with a velocity 5 m/s. It collides with a
stationary ball B, with a mass of 4 kg. After the collision, ball A moves in a
direction 60.0 degrees to the left of its original direction, while ball B moves in a
direction 50.0 degrees to the right of ball A's original direction. Calculate the
velocities of each ball after the collision.

Solution:
In the present problem we need to use the momentum conservation law: the net
momentum of two balls before the collision is equal to the net momentum of two
balls after the collision. It is important that momentum is a vector. Then the net
momentum of two balls is a vector sum of the momentum (vector) of ball A and
the momentum (vector) of ball B.
Therefore, the initial net momentum of two balls is
A momentum of a ball is a product of a mass of a ball and its velocity. Then

The final momentum is

Then the momentum conservation law takes the form


or

.......................(1)

120

To solve this vector equation we need to introduce coordinate system (axis x and
axis y) as shown in the figure.
We follow the standard procedure of finding the components of vectors and rewrite
the vector equation (1) in terms of x and y-components:
x-component of equation (1):

y-component of equation (1):

Finally, we have system of two equations with two unknown variables

121

From the second equation we obtain


We substitute this expression in the first equation:

Then

Therefore the velocities of the balls after collision are 22.9 m/s and 12.8 m/s (the
initial kinetic energy is less than the final energy, which means that during the
collision there is an additional source of energy).
68.

A skater of mass 80 kg initially at rest speeds up to a final speed of 10.0 m/s along
a straight line and towards the East direction.
(a) Find the momentum of the skater while at rest.
(b) Find the momentum of the skater while traveling with its final speed.
(c) Find the change in momentum of the skater.
(d) Find the impulse acted on the skater.
(e) If that impulse exerted on the skater acts for 4 s, find the average force acting
on the skater.

Solution:
In this problem we need to use the definition of momentum and impulse.
Momentum of an object is a vector, which is defined as the product of a mass of
the object and its velocity:
Therefore the direction of momentum is the same as the direction of velocity and
the magnitude of momentum is equal to the product of the mass of the object and
its speed (the magnitude of velocity).
(a) If the skater is at rest then its speed is zero. Then the momentum of the skater is
0.

(b) The final speed of the skater is10 m/s, its mass is 80 kg, then the final
momentum is

(c) The change in the momentum is equal to the difference between the final
momentum and initial momentum:

122

(d) The impulse acted on the skater is equal to the change of its momentum:

(e) At the same time he impulse acted on the skater is equal to the product of the
(average) force acted on the skater and the time
We know time: t 4 s . Then we can find the force:

69.

A toy car 1 of mass 0.30 kg moves along a frictionless surface with a velocity of
0.20 m/s. It collides with another toy car 2, with a mass of 0.40 kg and a speed
of 0.10 m/s in the same direction. After the collision, toy car 1 continues to
move in the same direction with a velocity of 0.15 m/s. Calculate the speed of toy
car 2 after the collision.

Solution:
We need to use the momentum conservation law: the net momentum of two toy
cars before the collision is equal to the net momentum of two cars after the
collision.
Then the net momentum of two cars is a vector sum of the momentum of car 1
and the momentum of car 2.
Therefore, the initial net momentum of two cars is
A momentum of a car is a product of a mass of a car and its velocity. Then

The final momentum is

Then the momentum conservation law takes the form


or

............................(1)

123

To solve this vector equation we need to introduce coordinate system. Since we


have motion along a line then it is enough to introduce only one axis: axis x (as
shown in the figure).
We follow the standard procedure of finding the components of vectors and rewrite
the vector equation (1) in terms of x-components:
x-component of equation (1):
Then

70.

A constant force of 80 N acts for 8 s on a box of mass 10 kg horizontally that


initially rests on a horizontal frictionless surface.
(a) Find the change in the box's momentum.
(b) Calculate the final speed of the box after the 8 s have passed.

Solution:

124

In this problem we need to use the definition of an impulse of a force and the
relation between the impulse and the chance of the momentum of the box.
If a constant force

acts on a box for a time

then the impulse of the force is

In the present problem a constant force of 80 N acts for 8 s. Then

and

. Then the impulse is

(a) The change in the box's momentum is equal to the impulse of the force acting
on the box:

(b) The initial speed of the box is zero. Then the initial momentum of the box is
zero. The change of the momentum is equal to the difference between the final
momentum and the initial momentum (which is zero). Then

Now we know the final momentum of the box. To find the final speed we just need
to divide the final momentum by the mass of the box:

71.

At an amusement park there is a roller coaster ride ("ride of Steel"). After the first
drop, riders are moving at the speed of 120 km/h, entering an underground tunnel.
Given the fact that the roller coaster was moving at a speed of 4 km/h at the top of
the hill, determine the vertical drop that these participants fell through. Neglect
friction.

Solution:
In the problem we need to use energy conservation law: since the friction is zero
then the mechanical energy, which is the sum of kinetic energy and gravitational
potential energy, is constant.
We need to use the correct SI units, which means that the speed should be
measured in m/s. Then

Now we need to write the energy conservation law. The initial mechanical energy
is the sum of initial potential energy and the initial kinetic energy:

The final energy is

125

The initial energy is equal to the final energy. Then

From the equation we obtain

We know the initial velocity, the final velocity, the initial height (it is zero). Then
we can find the final height (vertical drop):

72.

A man drops a 10 kg rock from the top of a ladder of height 5 m.


What is its speed just before it hits the ground?
What is its Kinetic Energy when it reaches the ground?

Solution:
In this problem we have free fall motion. We can solve this problem either by
using the kinematics equations, which describe the motion with constant
acceleration (free fall motion) or by using the energy conservation law.
Below we solve this problem by using the energy conservation law.
We assume that there is no friction, then we have the conservation of the net
mechanical energy of the rock. The mechanical energy of the rock is a sum of the
kinetic energy and gravitational potential energy:
The kinetic energy is determined by the speed of the rock:

The gravitational potential energy depends on the height of the rock:


Then the mechanical energy is

The conservation of the mechanical energy means that the mechanical energy in
the initial state is equal to the mechanical energy in the final state:

or

126

.......................................(1)
The initial velocity is zero:
the initial height is 5 m:
the final height is zero (ground):
We do not know the final velocity:

We substitute these values in the equation (1) and obtain

Now we can find the final kinetic energy:

73.

A cyclist intends to cycle up a 8 degrees hill whose vertical height is 150 m. If


each complete revolution of the pedals moves the bike 6 m along its path,
calculate the average force that must be exerted on the pedals tangent to their
circular path. Neglect work done by friction and other losses. The pedals turn in a
circle of diameter 30 cm. The total mass of the cyclist and his bike is 100 kg.

Solution:
In this problem we need to use generalized work-energy theorem: work done by an
external force is equal to the change of the net mechanical energy of the system:

We assume that the cyclist moves with constant speed. Then the initial and the
final kinetic energies are the same. Therefore in the above expression we need to
take into account only the gravitational potential energy:

where

is the total mass of the cyclist and bike.

Then

127

Now we need to define the initial and the final states of the system. We introduce
the final state as the state after one complete revolution (relative to the initial state).

We know that after one complete revolution the cyclist moves 6 meters (as shown
in the figure). Then the change in the height of the cyclist is
Therefore the change in the gravitation potential energy of the cyclist + bike is

The work done by the force is


During one revolution the pedal travels a distance of
(circumference of a circle with diameter d). Where d = 30 cm = 0.3 cm is the
diameter of the circle. Then the work done by the force is

This work is equal to the change in gravitational potential energy:


From this expression we can find the force:

74.

A body of mass 5 kg slides a distance of 6 m down a rough Inclined plane 30


degree. Then it moves on frictionless horizontal surface and compresses a spring.
The coefficient of kinetic friction is 0.1 and the spring constant is 300 N/m. Find
the maximum compression of the spring.

128

Solution:
In this problem we need to use the work-energy theorem, which determines the
relation between the work done by the friction force and the change of the net
mechanical energy.
According the work-energy theorem the work done by the friction force (which is
always negative) is equal to the difference between the final mechanical energy
and the initial mechanical energy.
................................................. (1)
The mechanical energy is the sum of kinetic energy, gravitational potential energy,
and elastic energy of the spring.
The initial velocity of the body is zero, the initial compression of the spring is zero,
the initial height of the body is
mechanical energy of the body is

. Then the initial

In the final state we have a maximum compression of the spring. It means that in
the final state the velocity of the body is zero. The final height of the body is 0
(ground level). Then the final mechanical energy of the body is

Then equation (1) takes the form


Now we need to find the work done by the friction force. The work done by the
friction force (see figure) is
where s = 6 m is the displacement of the body over incline plane. The friction force
is determined by the normal force and the coefficient of kinetic friction:
To find the normal force we need to write down the second Newton's law for the
motion along the incline. There are three forces acting on the body: gravitational
force, normal force, and the friction force. Then the second Newton's law takes the
form:
...........................................................(2)

129

The direction of the acceleration is along the incline. It means that the acceleration
has only x component (see figure). The y-component of acceleration is zero. Then
to find the normal force we need to write down only the y-component of equation
(2):
From this equation we can find the normal force:
Now we can find the friction force
and the work done by the friction force

Then we can find the maximum (final) compression of the spring:

75.

A uniform, solid metal disk of mass 6.0 kg and diameter 2.0 cm hangs in a
horizontal plane, supported at its center by a vertical metal wire. You find that it
requires a horizontal force of 4 N tangential to the rim of the disk to turn it by 5
degrees, thus twisting the wire. You now remove this force and release the disk
from rest.
(a) What is the torsion constant for the metal wire?
(b) Write the equation of motion for twist angle of the disk.

130

Solution:
(a) The torque is related to the torsion constant (k) by the equation:

We know that when we apply the force 4 N the system is in equilibrium with a
twist angle

. The torque in this case is


. Then

(b) To write equation of motion we need to find the moment of inertia of the disk.
It has the following expression:

Then
or

The tallest spot on Earth is Mt. Everest, which is 8857 m above sea level. If the
radius of the Earth to sea level is 6369 km, how much does the magnitude of g
change between sea level and the top of Mt. Everest?

Solution:
The value of the free fall acceleration is

Then

Or

131

76.

The value of g at the surface of the earth is 9.78 N/kg, and on the surface of Venus
the magnitude of g is 8.6 N/kg. A cosmonaut has a mass of 60 kg on the surface of
the earth. What will her weight be on the surface of Venus?

Solution:
The weight is the product of the mass of cosmonaut and the free fall acceleration.
On the surface of Venus:

77.

A car (m=2000 kg) is parked on a road that rises 20 degrees above the horizontal.
What are the magnitudes of (1) the normal force and (2) the static frictional force
that the ground exerts on the tires?

Solution:
We need to write down the second Newton's law: the net force is 0. This is the
vector equation. In the present problem there are three forces acting on the car:
gravitational force, normal force, and the static friction force.
Then we rewrite the second Newton's law in terms of x and y-components (x axis
is parallel to the road).
The x-component of the second Newton's law has the form:
The y-component:

From these equations we can find the normal force:


and the static friction force:

78.

A spaceship is on a straight-line path between Earth and moon. At what distance


from the Earth is net gravitational force on the spaceship zero?

Solution:
Let us introduce the mass of the Earth as M earth , the mass of the moon as M moon ,
and the distance between the Earth and the moon as R .
If the spaceship is at distance x from the Earth, then the gravitation force on the
spaceship due to Earth is

The gravitation force on the spaceship due to moon is

132

Since the net gravitation force should be zero, then we have the equation:

From this equation we have:

and

79.

The space shuttle is orbiting the Earth at a distance of about 300 km from its
surface. At that distance, the gravitational acceleration is almost the same as that
on the surface. How long does it take for the shuttle to complete one orbit around
the Earth? Assume that the orbit is circular.

Solution:
The gravitation force will provide the acceleration to the shuttle, which is equal

where
Then a g and

Then the period is

80.

When a falling object is at a distance above the Earth's surface of 4 times the
Earth's radius, what is its free-fall acceleration due to the Earth's gravitational force
exerted on it?

Solution:
The gravitation force has the following expression

133

Here M is the mass of the Earth, R is the radius of the Earth, h is the distance
above the Earth. Since h 4R , then

Then acceleration is

Since g G

M
9.8 m / s 2 , then
2
R

The drawing shows three particles far away from any other objects and located on
a straight line. The masses of these particles are: particle A = 400 kg, particle B =
500 kg, and particle C = 100 kg. Find the magnitude and direction of the net
gravitational force acting on each of the three particles (the direction to the right is
positive).

Solution:
Particle A: The net gravitational force is the sum of two forces: due to particle B
and due to particle C. Both forces have positive direction:

Particle B: The net gravitational force is the sum of two forces: due to particle A
(this force is negative, which means that its direction is to the left) and due to
particle C (this force is positive). Then

134

Particle C: The net gravitational force is the sum of two forces: due to particle A
and due to particle B. Both forces are negative, then

135

Potrebbero piacerti anche